Vous êtes sur la page 1sur 67

Risk and Insurance 1 Risk Management & Insurance Planning 1.

Composite Insurance Agent Holds a license To act as an Insurance agent for a a. Life Insurer b. General Insurer c. Both of the above. d. None of the Above. Solution: c A composite agent is one who is authorized to sell both Life & General insurance products. 2. In property insurance (not in marine insurance) insurable interest must exist at the time of a. Loss b. Contract c. Both a & b d. None of the above Solution: c In property insurance one needs to have insurable interest both at the time of contract and loss. Exception is Marine insurance were in one needs to have insurable interest only at the time of Loss. Further in Life insurance one needs to have insurable interest only at the time of contract. 3. Gambling is an example of a. Pure risk. b. Speculative risk c. Personal risk d. Property risk Solution: b 4. Which of the following is the third step in identifying risks a. Identify risks b. Develop Alternative for handling risks c. Choose and implement an appropriate Strategy. d. Analyze and evaluate risks (Risk Measurements) Solution: b 5. Pure risk can be defined as a situation where a. there is a chance of loss or no loss b. There is a chance of a loss or a profit c. There is a no chance of loss d. All of the above. Solution: a 6. An organization cannot finance its own risk cost (risk financing ) a. Payment out of current expenses b. By debt or equity financing c. By pre or post credit. d. By approaching a reinsurance company. Solution: d 7. _______________ is the right of one person, having indemnified another under a legal obligation to do so, to stand in the place of that other and avail himself of all the right and remedies of that other, whether already enforced or not. a. Substitution. b. Indemnity c. Adhesion d. Subrogation. Solution: d 8. Touching another person unlawfully is an example of a. Intentional Tort. b. Absolute liability. c. Negligence. d. Conclusive tort. Solution: a Tort is civil wrong doing, which could be divided in to 3 types (1) Intentional, (2) Absolute and Strict liability and (3) Negligence. 9. Pricing of a product in an Life insurance company is a function of a. Marketing b. Underwriting c. Actuarial d. Finance

Solution: c 10. Collection and payments of monies in an Life insurance company is a function of a. Marketing b. Actuarial c. Finance d. Legal and secretarial Solution: c 11. Auditing in an Life insurance company is a function of a. Underwriting b. Actuarial c. Finance d. Legal and secretarial Solution: c 12. Compliance to regulator and other public supervisory bodies in an Life insurance company is a function of a. Underwriting b. Actuarial c. Finance d. Legal and secretarial Solution: d 13. The amount of insurance accepted by the re-insurer is known as a. Premium b. Net premium c. Cession. d. Retention. Solution: c 14. Which policy meets the need of security for childrens education the best a. Term assurance plan b. Children deferred assurance plan c. Deferred annuity plan d. Health insurance. Solution: b 15. Re-insurance Advisory committee consist of ____________ persons a. Five b. Four c. Six d. Ten Solution: a 16. Insurer has a period of within _________ days of all reports received to decide about the claim. a. 30 days b. 45 days c. 60 days d. 90 days Solution: a 17. For any applicant to be registered as an IRDA licensed agent, the minimum qualification required in an urban area is a. X th standard b. XII th standard c. Graduation. d. Post-Graduation. Solution: b 8. Employees liability for his employees injury even when there is no negligence on the part of the employer is an example of a. Intentional Tort. b. Absolute liability. c. Negligence. d. Conclusive tort. Solution: b In case of strict liability certain defenses are allowed but not so in Absolute liability. 19. Investigation regarding mortality factor in an Life insurance company is a function of a. Marketing b. Underwriting c. Actuarial d. Legal and secretarial

Solution: c 20. The limit of indemnity for a Direct Broker is A] Three times remuneration received at the end of every financial year subject to a minimum limit of rupees fifty lakhs. B] Three times remuneration received at the end of every financial year subject to a minimum limit of rupees Two crores and fifty lakhs. C]Three times remuneration received at the end of every financial year subject to a minimum limit of rupees Five crores. D]Three times remuneration received at the end of every financial year subject to a minimum limit of rupees Ten crores. Solution: a Professional Indemnity requirement as per IRDA norms : Direct broker - 3 times of remuneration in PY subject to Min- 50 lakhs Reinsurance - 3 times of remuneration in PY subject to Min- 2.5 cr. Composite -3 times of remuneration subject to with min 5 cr. 20. An ultimate mortality table excludes data for . first 5 to 15 years . first 3 to 5 years . first 3 to 6 years . first 7 to 15 years Solution: a 20. The cheapest policy in the market is . A term policy . A unit linked policy . An endowment policy . A Whole life policy Solution: a 20. Participating polices are policies entitled to . risk sharing . premium . bonus . fixed payments Solution: c 20. An increasing term insurance means . The term insurance sum assured increases with inflation . Both the premium and sum assured increases . The term insurance increases at intervals . Only the premium keeps on increasing. Solution:c 20. Overseas mediclaim policy does not cover one of the following : . Loss of passort . Loss of baggage . Medical accidents . Personal accident. Solution: b It is loss of checked in baggage which is covered. 20. ________________ insurance is a Tariff Product . life . motor . health . marine Solution: b Now only Third party is tariffed and not motor. 20. The process of due diligence conducted by an insurance agent is known as . Underwriting . Investigation . Inspection . Site Check. Solution: a 20. This type of insurance is not easily available in India . Officers liability . Disability Income Protection. . Health

. Life Solution: b 20. Insurance Advisors and Agents are governed by . IRDA . SEBI . AMFI . Both IRDA & SEBI. Solution: a 20. Mortality rate is . Rate of death in a group. . Rate of death in the population. . Probability of death in a group . Probability that the insured will die before reaching a certain age. Solution: d 20. HLV explains in general . Total income of an individual . Total worth of an individual. . Total wealth of an individual. . Earning Capacity. Solution: c 20. Adverse loss ratio refers to . Higher premium being received by the insurer . Lower premium being received by the insurer . Lower than expected claim ratio in the pool . Higher than expected claims ratio in the pool. Solution: d 20. The other name of no fault liability is . Absolute Liability. . Strict liability. . Vicarious Liability . Contractual Liability. Solution: a 20. The limit of indemnity for a Reinsurance Broker is . Three times remuneration received at the end of every financial year subject to a minimum limit of rupees fifty lakhs. . Three times remuneration received at the end of every financial year subject to a minimum limit of rupees Two crores and fifty lakhs. . Three times remuneration received at the end of every financial year subject to a minimum limit of rupees Five crores. . Three times remuneration received at the end of every financial year subject to a minimum limit of rupees Ten crores. Solution: b 20. Which policy meets the need of Security after retirement the best . Term assurance plan . Children deferred assurance plan . Deferred annuity plan . Health insurance. Solution: c 20. A sub standard Indian life is considered . an individual with health problems . a below average intelligent Indian life. . A life which cannot be insured . Depends on the underwriting standards of the insurance company. Solution: d 20. In insurance, the contingency insured against is usually the . Financial loss . Certain financial loss . Loss of profit . Uncertain financial loss. Solution: d 20. Insurance companies often offer the services of specialists and experts to industry in different fields. This is for rendering advice on . Reduction in uncertainty

. Loss prevention/minimization . Indemnification . None of the above. Solution: b 20. Which policy meets the need of Security of the family the best . Term assurance plan . Children deferred assurance plan . Deferred annuity plan . Health insurance. Solution: a 20. One of the following is not a valid exclusion for Temporary Total Disablement . Any existing disablement . Disablement of parent . Insanity . War risk. Solution: b 20. The limit of indemnity for a Composite Broker is . Three times remuneration received at the end of every financial year subject to a minimum fifty lakhs. . Three times remuneration received at the end of every financial year subject to a minimum Two crores and fifty lakhs. . Three times remuneration received at the end of every financial year subject to a minimum Five crores. . Three times remuneration received at the end of every financial year subject to a minimum Ten crores. Solution: c 20. The disadvantage of a term policy . it is expensive . it is only for a short period . there is no bonus . All of the above. Solution: c 20. Which policy meets the need of special needs the best . Term assurance plan . Children deferred assurance plan . Deferred annuity plan . Health insurance. Solution: d 20. Market Research of existing products in an Life insurance company is a function of . Marketing . Underwriting . Actuarial . Finance Solution: a 20. One of the following is not a valid exclusion for Critical Illness Insurance . HIV virus infection. . Suicide or self inflicted. . Pre-existing Critical illness. . Existing disability. Solution: d 20. The principle of subrogation is derived form the principle of . Utmost good faith . Adhesion . Duty of disclosure . Indemnity Solution: d 20. The duty of disclosure follows from . The doctrine of adhesion . The principal of subrogation . The principle of indemnity . The principle of utmost good faith Solution: d 20. Offer in life insurance contract refers to

limit of rupees limit of rupees limit of rupees limit of rupees

. Proposer paying first premium . Proposers application form for insurance . Original policy document . Companys brochure duly authenticated. Solution: b 20. Health insurance for children is provided for children aged 4 years . With no precondition . One of the parents being insured. . Both the parents covered concurrently . Both A and B Solution: c 20. Domicillary hospitalization period is defined as . One day . Two days . Three days . Four days. Solution: c 20. Jan Arogya Bima policy is a . Life insurance Policy . Marine insurance policy . Travel insurance policy . Health insurance. Solution: d Jan Arogya policy is a Medicalim / health insurance policy by United India Insurance Company. 20. Publication of a false statement in a permanent form designed to damage the reputation of another person. Is an example of . Intentional Tort. . Absolute liability. . Negligence. . Conclusive tort. Solution: a Intentional Tort could be Libel - Publication of false statement to damage others reputation. Slender - Damaging reputation verbally Assault - Unlawfully touching another person. 20. Bhavishya Arogya policy is meant for persons in the age group between . 25 to 55 years . 30 to 55 years . 25 to 60 years . 30 to 60 years. Solution: a Bhavishya Arogya is a life term policy where medical benefits are made available after retirement of the insured- by New India Assurance. 20. The disadvantage of a whole life policy . it is expensive . it is only for a short period . there is no bonus . All of the above. Solution: a 57. In insurance parlance TPA means . Third party Assessor . Third Party Adjustor . Third Party Administrator . Total Premium Amount Solution: c 58. The working capital of a TPA should be minimum . Rs. 5,00,000 . Rs. 10,00,000 . Rs. 5,00,000 . Rs. 10,000,000 Solution: d 59. Commercial vehicle insurance does not include . Goods carrying vehicle insurance. . Passenger carrying vehicle insurance. . A family car insurance . All of the above. Solution: c 60. In life insurance insurable interest must exist at the time of . Loss . Contract . Both a & B . None of the above

Solution: b n property insurance one needs to have insurable interest both at the time of contract and loss .Exception is Marine insurance were in one needs to have insurable interest only at the time of Loss. Further in Life insurance one needs to have insurable interest only at the time of contract. 61. In insurance market Dynamics, the risk manager must choose between . Risk evaluation & speculation. . Risk rentention and risk transfer . Risk identification and safety . Risk financing and actuarial aspects. Solution: a 62. The insurance company has to respond to all communication received from the policyholders within . A week of receipt of such communication . 15 days of their receipt . 10 days of their receipt . 20 days of their receipt. Solution: c 63. In hazard logic tree, under the peril water damage, hazards are . Malicious Damage, subsidence, hurricane . Flood, storm, bust pipe or other apparatus, burst water main . Spillage or leakage or Flammable Liquids. . Subsidence and flooding. Solution: b All are water related perils. 64. Which of the following is not a classification of term insurance policy . sum assured with bonus return term insurance . Level benefit term assurance . Increasing benefit term assurance . Decreasing benefit term insurance Solution: a 65. Particular risk is a risk that affects . Entire economy . Only individual . Government . None of the above Solution: b Solution: Particular risk effects an individual where as Fundamental risk effects whole economy. 66. Home loan/Mortgage protection insurance refers to . Property insurance policy . Credit insurance policy . Life insurance policy guaranteeing the mortgage loan. . None of the above Solution: c 67. The most basic risk management decision is . Whether to provide fire protection equipment . Perhaps to explore, whether to purchase insurance . Analyze the probability distribution . None of the above Solution: b 68. It is compulsory for the broker to maintain . Liquidity margin . Cash margin . Solvency margin . Cash balance Solution: c 69. While treating the appropriate techniques for treating loss exposures, the major issue for top risk management concern is . Disability management . Regulatory changes . Occupational Safety and health Act regulations . Workers compensation costs. Solution: d 70. In controlled Third party captive business, retailers are underwriting . Credit life, health and disability; extended warranty . Subcontractor coverages . Joint ventures . Credit life, health and disability; mortgage insurance, title insurance, auto and property. Solution: a 71. To keep the license in force the composite broker shall have to pay registration fees calculated at the rate of ____________ per annum. . Rs. 2 Lakhs . Rs. 3 Lakhs . Rs. 25,000

. Rs. 50,000 Solution: b 72. Quick asset ratio is used to a. Determine the investment capacity of the firm a. Determine firms liquidity to cover unexpected payment too. a. Determine the current assets of the firm a. Determine the current liabilities of the firm. Solution: b 73. Standard deviation and coefficient of variation d. Are used in fair value accounting d. Economic indicators d. Methods used to measure dispersions d. None of the above. Solution: c 74. Product development in an Life insurance company is a function of d. Marketing d. Underwriting d. Actuarial d. Finance Solution: a 75. Self insurance funds may be identified as d. Contingency funds used to spread the cost of losses more evenly over a large term d. Profitability not due to normal trading, but because of exceptionally good or bad loss experience d. Contribution collected from each of the division of the company d. None of above. Solution: a 76. If unacceptable Risk is accepted d. This would bankrupt the firm d. Outcome will be that of a speculative risk d. Outcome will be that of a dynamic risk d. None of the above. Solution: a 77. Charging of losses to current operating costs d. Is a form of reducing losses. d. Is a form of evaluating the losses. d. Is a form of financing the losses. d. Is a form of Identifying the losses. Solution: c 78. The waiting period under a disability benefit policy refers to d. The period of time that must elapse before the policy benefit will commence d. The period of time that must elapse before settlement of the claim after submission. d. The period of time that must elapse before the issue of the policy document. d. The period of time that must elapse before the proposal can be considered. Solution: a 79. The law of reinsurance is primarily based on d. law of torts d. law of vicarious liability d. law of contract d. law of specific relief Solution: c 80. An Indian Insurance company can transact d. Life insurance and general insurance business d. Life insurance and reinsurance business d. General and reinsurance business d. Either Life or general insurance or reinsurance business. Solution: d 81. The reinsurance contract itself may relate to d. One particular company d. One particular ceding insurer d. One particular insurance d. One particular catastrophe Solution: c 82. The following factor is not very important form an underwriters view d. Age d. Sex d. Occupation d. Family income Solution: d 83. In fire insurance, a reinsurance policy was called as d. A guarantee policy but nowadays is more commonly referred to as facultative reinsurance d. A coagulative policy on property insurances with specific focus on catastrophe insurances d. A fire fighting comprehensive policy with focus on mega risks

d. An extraneous perils covering policy in the event of disaster. Solution: a 84. Ram insures his home worth Rs.50 lakhs for Rs.30 lakhs. The house is destroyed in a fire and he suffers losses worth Rs.20 lakhs. How much will he receive from the insurance company? d. Rs.10 lakhs d. Rs.20 lakhs d. Rs.16 lakhs d. Rs.12 lakhs Solution: [a] The answer is Rs. 10 lakhs because Ram is underinsured by 40%. 85. Reinsurance cannot exist d. Without its global character d. Without captive insurers d. Without there being direct insurance, for the first party is reinsured who has already issued a direct policy d. Without past data of catastrophe risk covered. Solution: c 86. The application of the law of contract does not apply to _____________contracts. d. Insurance d. Stock markets d. Property deals d. None of them Solution: d 87. Every reinsurance instrument executed out of India chargeable with duty may be stamped d. Within four months d. Within five months d. Within three months d. Within six months. Solution: c 88. Insurance can be ________contracts. A = Benefit, B= Indemnity, C= Negotiated. d. Only A d. Either B or C d. Only A d. Only B Solution: b 89. Fundamental risk is a risk that affects d. Entire economy d. Only individual d. Government d. None of the above Solution: [a] Fundamental risk effects whole economy and particular risk effects individual. 90. Which one of the following is not a personal risk d. risk of premature death d. risk of sickness d. risk of unemployment d. risk of car accident Solution: d 91. Policy cost/conversion = 0.001, Interest allowed equivalent to after tax return =20%, annual premium = 13,000, Dividend or bonus = 23,000 death benefits = 20,00,000 Cash surrender value at the end of current policy period=6,50,000 , cash surrender value at the end of previous policy year = 570000. What is policy cost per thousand? d. 19.70 d. 44.66 d. 57.84 d. 122.51 Solution: [a] Policy Cost (per thousand) = [PM + CSVP] [1+i]-[CSV + D] ---------------------------------------[DB-CSV] * 0.001 PM= Premium-yearly; CSV = Cash surrender value at the end of policy year; CSVP = Cash surrender value at the end of previous year; i = interest or rate of return; D =Dividend or bonus; DB = Death benefit; policy cost conversion given=0.001. Hence answer is 19.70. 92. Policy cost/conversion = 0.001, Interest allowed equivalent to after tax return =19%, annual premium = 14,000, Dividend or bonus = 22,000 death benefits = 25,00,000 Cash surrender value at the end of current policy period=6,00,000 , cash surrender value at the end of previous policy year = 580000. What is policy cost per thousand? d. 19.70 d. 44.66 d. 57.84 d. 122.51

Solution: {b}

Policy Cost (per thousand) = [PM + CSVP] [1+i]-[CSV + D] ---------------------------------------[DB-CSV] * 0.001 PM= Premium-yearly; CSV = Cash surrender value at the end of policy year; CSVP = cash surrender value at the end of previous year; i = interest or rate of return; D = Dividend or bonus; DB = Death benefit; policy cost conversion given=0.001. Hence answer is 44.66. 93. Policy cost/conversion = 0.001, Interest allowed equivalent to after tax return =18%, annual premium = 14,000, Dividend or bonus = 21,000 death benefits = 30,00,000 Cash surrender value at the end of current policy period=5,50,000 , cash surrender value at the end of previous policy year = 590000. What is policy cost per thousand? d. 19.70 d. 44.66 d. 57.84 d. 122.51 Solution: {c} Policy Cost (per thousand) = [PM + CSVP] [1+i]-[CSV + D] ---------------------------------------DB-CSV] * 0.001 PM= Premium-yearly; CSV = Cash surrender value at the end of policy year; CSVP = cash surrender value at the end of previous year; i = interest or rate of return; D =Dividend or bonus; DB = Death benefit; policy cost conversion given=0.001. Henceanswer is 57.84. 94. Policy cost/conversion = 0.001 , Interest allowed equivalent to after tax return =18% annual premium = 15000, Dividend or bonus = 20000 death benefits = 20,00,000 Cash surrender value at the end of current policy period=525,000 , cash surrender value at the end of previous policy year = 600,000. What is policy cost per thousand? d. 19.70 d. 44.66 d. 57.84 d. 122.51 Solution: {d} Policy Cost (per thousand) = [PM + CSVP] [1+i]-[CSV + D] ---------------------------------------[DB-CSV] * 0.002 PM= Premium-yearly; CSV = Cash surrender value at the end of policy year; CSVP = cash surrender value at the end of previous year; i = interest or rate of return; D =Dividend or bonus; DB = Death benefit; policy cost conversion given=0.002. Hence answer is 122.51 95. Policy cost/conversion = 0.002, Interest allowed equivalent to after tax return =20%, annual premium = 13,000, Dividend or bonus = 23,000 death benefits = 20,00,000 Cash surrender value at the end of current policy period=6,50,000 , cash surrender value at the end of previous policy year = 570000. What is policy cost per thousand? d. 9.85 d. 22.33 d. 28.92 d. 61.25 Solution: {a} Policy Cost (per thousand) = [PM + CSVP] [1+i]-[CSV + D] ---------------------------------------[DB-CSV] * 0.002 PM= Premium-yearly; CSV = Cash surrender value at the end of policy year; CSVP = cash surrender value at the end of previous year; i = interest or rate of return; D =Dividend or bonus; DB = Death benefit; policy cost conversion given=0.002. Hence the answer is 9.85 96. Policy cost/conversion = 0.002 , Interest allowed equivalent to after tax return =19%,annual premium = 14,000, Dividend or bonus = 22,000 death benefits = 25,00,000 Cash surrender value at the end of current policy period=6,00,000 , cash surrender value at the end of previous policy year = 580000. What is policy cost per thousand? d. 9.85 d. 22.33 d. 28.92 d. 61.25 Solution: {b} Policy Cost (per thousand) = [PM + CSVP] [1+i]-[CSV + D] ---------------------------------------[DB-CSV] * 0.002 PM= Premium-yearly; CSV = Cash surrender value at the end of policy year; CSVP = cash surrender value at the end of previous year; i = interest or rate of return; D =dividend or bonus; DB = Death benefit; policy cost conversion given=0.002. Hence the answer is 22.33 97. Policy cost/conversion = 0.002 , Interest allowed equivalent to after tax return =18%, annual premium = 14,000, Dividend or bonus = 21,000 death benefits = 30,00,000 Cash surrender value at the end of current

policy period=5,50,000 , cash surrender value at the end of previous policy year = 590000. What is policy cost per thousand? d. 9.85 d. 22.33 d. 28.92 d. 61.25 Solution: {c} Policy Cost (per thousand) = [PM + CSVP] [1+i]-[CSV + D] ---------------------------------------[DB-CSV] * 0.002 PM= Premium-yearly; CSV = Cash surrender value at the end of policy year; CSVP = cash surrender value at the end of previous year; i = interest or rate of return; D =Dividend or bonus; DB = Death benefit; policy cost conversion given=0.002. Hence the answer is 28.92 98. Policy cost/conversion = 0.002 , Interest allowed equivalent to after tax return =18% annual premium = 15000, Dividend or bonus = 20000 death benefits = 20,00,000 Cash surrender value at the end of current policy period=525,000 , cash surrender value at the end of previous policy year = 600,000. What is policy cost per thousand? d. 9.85 d. 22.33 d. 28.92 d. 61.25 Solution: {d} Mortality = 30000/5000000 = 0.006 No. of people who would die = 50000 * 0.006 = 300 Amount required to pay them = 300 * 200000 = 6 crores Therefore premium required = 6,00,000,00 / 50000 = 1200 Hence the answer is 61.25 99. A group of 50000 persons each aged 35 years wishes to apply for term insurance for a one year period for a sum of Rs.2,00,000. If mortality tables show that out of 50,00,000 people 30,000 die within a year, find the premium to be paid by each of the 50000 applicants. d. 1200 d. 1250 d. 1300 d. 1400 Solution: a 100. A group of 75000 persons each aged 33 years wishes to apply for term insurance for a one year period for a sum of Rs.2,50,000. If mortality tables show that out of 100,00,000 people 25,000 die within a year, find the premium to be paid by each of the 75000 applicants. d. 625 d. 700 d. 750 d. 800 Solution: a 101. A group of 45000 persons each aged 34 years wishes to apply for term insurance for a one year period for a sum of Rs.3,50,000. If mortality tables show that out of 60,00,000 people 30,000 die within a year, find the premium to be paid by each of the 45000 applicants. d. 1650 d. 1700 d. 1750 d. 1800 Solution: c 102. A group of 55000 persons each aged 40 years wishes to apply for term insurance for a one year period for a sum of Rs.3,00,000. If mortality tables show that out of 75,00,000 people 50,000 die within a year, find the premium to be paid by each of the 55000 applicants. d. 1800 d. 1900 d. 2000 d. 2100 Solution: c 103. A group of 35000 persons each aged 30 years wishes to apply for term insurance for a one year period for a sum of Rs.3,00,000. If mortality tables show that out of 120,00,000 people 60,000 die within a year, find the premium to be paid by each of the 35000 applicants. d. 1500 d. 1250 d. 1400 d. 2750 Solution: a 104. A group of 50000 persons each aged 28 years wishes to apply for term insurance for a one year period for a sum of Rs.2,50,000. If mortality tables show that out of 50,00,000 people 25,000 die within a year, find the premium to be paid by each of the 50000 applicants. d. 1500 d. 1250 d. 1400 d. 2750 Solution: b

105. A group of 60000 persons each aged 45 years wishes to apply for term insurance for a one year period for a sum of Rs.2,80,000. If mortality tables show that out of 80,00,000 people 40,000 die within a year, find the premium to be paid by each of the 60000 applicants. d. 1500 d. 1250 d. 1400 d. 2750 Solution: c 106. A group of 55000 persons each aged 40 years wishes to apply for term insurance for a one year period for a sum of Rs.3,00,000. If mortality tables show that out of 60,00,000 people 55,000 die within a year, find the premium to be paid by each of the 55000 applicants. d. 1500 d. 1250 d. 1400 d. 2750 Solution: d 107. Calculate HLV to recommend adequate insurance cover; Mr. Ritesh, Age= 30 yrs Retirement age 60 years. He is Asst. Vice president. His monthly salary is 55,000. he pays professional tax of rs.3000 and income tax subject to allowable deductions i.e. tax paid Rs. 132000 Reasonable self maintenance expenditure estimated Rs.45000 p.a.; life insurance premium for self Rs. 18000 with total sum assured Rs. 1200000. For wife and child he pays insurance premium of Rs. 10500 and Rs. 6500 respectively rate of interest assumed for capitalization of future income is at 8%. Adequate additional insurance recommended is d. Rs. 50,00,000 d. Rs. 42,00,000 d. Rs. 55,00,000 d. Rs. 10,000,000. Solution: b 108. Calculate the HLV to recommend total insurance cover Mr. APTE. His present age 45 years. His retirement age is 70 yrs. His total income of Rs.21,00,000 ( Salaries 15 lakhs, dividends on individual shares 2.5 lakhs, HUF co-partner profit share 1.5 lakhs, interest on bank deposit and other investment 1 lakh, minor daughters income added 1 lakh.) he has paid following taxes: Corporate professional tax 5000 income tax 410000 as per individual tax return filed. He pays total life insurance premium of Rs.55000 (self insurance cover S.A. 22,00,000) Reasonable maintenance charge for a person of his stature is assumed as Rs. 100000p.a. applied rate of interest to arrive at a present value of his future income is at 5 % d. Rs. 2.26 crores d. Rs. 3 crores d. Rs. 1.80 crores d. Rs. 12 crores. Solution: a 109. Mr. and Mrs. Rao, aged 46 and 42 years, both have a life expectancy of 35 years. Calculate the insurance required based on need based and income replacement methods on Mr. Raos life. You have the following information Current investments: Rs.25,00,000, Expenses Rs. 3,00,000 (including 1 lakh of Mr. Raos personal Expenses) Mr. Raos income post tax rs. 3.5 lakhs. Final costs Rs. 1 lakhs. Post tax, post inflation rate/discount factor is 3%. d. 42 lakhs, 117 lakhs d. 20 lakhs, 68 lakhs d. 42 lakhs,83 lakhs d. 20 lakhs, 52 lakhs. Solution: c 110. A planner and prospect agreed for an endowment policy for Rs.4 lakhs limited payment for 10 years with a term of 20 years. If reversionary bonus is taken as 7.5 % per annum and terminal bonus as Rs. 150 per 1000 what will be maturity value? d. Rs.10,50,000 d. Rs.10,60,000 d. Rs.10,70,000 d. Rs.10,80,000 Solution: [b] Revisionary Bonus = 30,000 * 20 = 6.00 lakhs Terminal Bonus 0.60 lakhs SA 4.00 lakhs ---------------Maturity value 10.60 lakhs 111. This is not a risk faced by the individual in their various activities d. Financial risk d. Cumulative risk d. Pure risk d. Speculative risk Solution: b 112. A person goes to the neighborhood lottery store and buys a lottery ticket. This is an example of

d. Pure risk d. Dynamic risk d. Speculative risk d. Non financial risk Solution: c 113. A living person will face the risk of death at all points of their life and this risk will be classified as d. Personal risk d. Property risk d. Liability risk d. None of the above Solution: a 114. Every time the children move out to play in the afternoon in the summer holidays their mother insists that they wear a cap to keep out the impact of the direct rays of the sun and to drink water to avoid dehydration. This is an example of d. Risk transfer d. Risk avoidance d. Risk retention d. Risk reduction Solution: d 115. A footballer who is voted as the European Footballer of the year does not take an insurance policy on his legs and continues to play as before. This is a case of d. Risk transfer d. Risk avoidance d. Risk retention d. Risk reduction Solution: c 116. While deciding upon the risks to be insured the various factors considered for the process are d. Various events of risk d. Severity of the financial loss d. Frequency of occurrence d. All of the above Solution: d 117. All the risks present for a person can be insured d. True because risk can be transferred d. False because it has to fulfill the required conditions of measurability, homogeneity etc d. True because a person can take the necessary insurance d. False because everyone does not have all the different kinds of risk Solution: b 118. An insurance contract is meant for the purpose of d. Indemnity d. Profit d. Capital gains d. Earning return Solution: (a) Indemnity is to bring back the insured back to the position he/she was prior to the loss. There should be no gain. 119. Insurance helps a person to d. Prevent a profit d. Prevent a loss d. Make a capital gain d. Earn a rate of return Solution: b 120. The role of insurance in the economy will relate to the process of d. Providing funds for investment and credit d. Controlling inflation d. Pushing up interest rates d. Controlling the foreign exchange Solution: (a) Insurance companies can provide long term funds for the country which can be channeled in to long term infrastructure projects 121. A person wants insurance for his motorcar. The point regarding the colour of his wall in his house will be a ________ point for the purpose of insurance d. Material d. Slightly material d. Non material d. None of the above Solution: c Material fact is any fact which would influence the insurer in accepting or declining a risk or in fixing the premium or terms and conditions of the contract. Here the colour of wall is non material to the risk being insured. 122. Under the principle of insurable interest there should be _______ that can be insured . Property

. Life . Potential liability . All of the above Solution: d 123. The following will not be factor covered for being insured under the principle of insurable interest . Right . Space . Interest . Limb Solution: b 124. The insured must have a _______ with the subject matter of insurance for benefiting from it ii. Right ii. Interest ii. Relationship ii. None of the above Solution: c 125. A person owns a building in which he stays. In this case insurable interest is created by d. Nature d. Common law d. Contract d. Statue Solution: d 126. A person has ______ insurable interest in his own life d. Part d. Specific d. Unlimited d. Limited Solution: c 127. A person got married in the last week of December 2005. In the month of November 2006 does the wife have an insurable interest in the life of the husband? d. No because a year has not passed since marriage d. Yes because a person has an automatic insurable interest in the life of the spouse d. No because only the husband has an insurable interest in the life of the wife d. None of the above Solution: b Risk and Insurance 19 128. In case of a mortgage the situation for the two parties in terms of insurable interest is d. Only the mortgagor has insurable interest d. Only the mortgagee has insurable interest d. Both have insurable interest arising d. None of the two have insurable interest Solution: c 129. The following will have insurable interest in the role that they play d. Trustees d. Bailee d. Executors d. All of the above Solution: d 130. A person is not supposed to profit from an insurance policy but should be in a position similar to what he was in before the loss occurred. This is outlined by the principle of d. Utmost good faith d. Insurable Interest d. Indemnity d. Subrogation Solution: c 131. The feature of indemnity can be undertaken through the route of d. Repairs d. Reinstatement d. Replacement d. All of the above Solution: d 132. The risk that are covered by an insurance policy are called d. Uninsured perils d. Insured perils d. Excluded perils d. None of the above Solution: b 133. A life insurance policy says that the death on account of a war will not be covered by the insurance policy. This will be classified as a d. Uninsured peril d. Insured peril d. Excluded peril d. None of the above Solution: c 134. In an insurance contract the side making the offer is

d. The insurance company d. The insured d. The beneficiary d. All of the above Solution: (b)In insurance contract , the insured makes offer and it is the company which accepts/denies. Further, if insurance company changes any terms it makes an counter offer to the insured and then it is up to insured to accept or deny. 134. In terms of competence from the side of the insurance company it has to d. Be over 18 years in existence d. Be over 5 years in existence d. Has to have an insurance license under the regulations d. All of the above Solution: c Risk and Insurance 20 135. The consideration in the insurance contract is the payment of d. Commission d. Premium d. Underwriting amount d. Fixed amount of payment Solution: b 136. The value exchanged in terms of premium paid and the payout in case of a loss is not equal but is based upon an uncertain event. This is based upon the feature of d. Aleatory contract d. Fixed contract d. Adhesion d. Utmost good faith Solution: a 137. An insurance company leaves the setting of the insurance rate upon the person who is assigned to set the rate. This person will look at the specific case and determine the premium rate. This is an example of d. Individual rating d. Class rating d. Merit rating d. None of the above Solution: a 138. This is not a way of determining the merit rating for an insurance company d. Schedule rating d. Experience rating d. Class rating d. Retrospective rating Solution: c Rating could be-1. Individual or judgmental. 2. Class or manual. 3. Merit rating Schedule , experience & Retrospective. 139. All of the following statements describe the operation of a life annuity EXCEPT: d. Because of the interest factor, an annuitant is assured of receiving back more than he or she paid in d. The annuitant is assured that he or she cannot outlive the length of time of the annuity payments d. The emphasis is on the liquidation of the fund as opposed to its growth d. The older the annuitant is when he or she receives the first annuity payment; the greater will be the amount of each payment Solution: a 140. A type of risk with high frequency but low severity is probably best handled by a. Avoidance b. Subrogation c. Self-insurance d. Under-Insurance Solution: c 142. Speculative risk can have following outcomes ________. (A) Loss (B) Gain (C) Status Quo d. A only d. B only d. C only d. A, B & C Solution: d 143. LALGI is ___________. d. Private contribution guarantee scheme d. Private insurance d. Public benefit guarantee scheme d. Social insurance Solution: (d) Landless Agricultural Labourers Group Insurance ( LALGI ) scheme is a social security group scheme where in head of the family betwen 18 to 60 years is eligible if has no land.On death till 60years a sum of Rs. 2000/is paid to the family.

144. For risk to be insurable which one of following is not correct? d. The loss must be fortuitous or accidental. d. The loss must not be catastrophic. d. The loss produced by the risk must be definite and measurable. d. There must be a sufficiently large number of heterogeneous exposure units to make the losses reasonably predictable. Solution: d The large number of units have to be homogenous. 145. In Unit-Link policy, market risk is with ______. (A) Insured (B) Insurer d. A d. B d. Both A & B d. Neither A nor B Solution: a 146. Minimum age at entry for Medi-claim is ___________. d. 8 years d. 91 days d. 365 days d. 18 years Solution: b Questions 147 to 182 carry two mark each 147. An insurance company pays the insured the sum for the goods destroyed in his godown. There is still some additional part of the goods that is picked up from the godown and this results in an additional sum of Rs 5000 coming in. In this situation the share of the insurance company is d. Rs 0 d. Rs 2500 d. Rs 5000 d. Rs 10000 Solution: c 148. There is a godown that is worth Rs 20 lakh. A person insures it for Rs 15 lakh and there is a fire that completely destroys the godown. In this case the insurance company will pay d. Rs 500000 d. Rs 1000000 d. Rs 1125000 d. Rs 1500000 Solution: c The property is under insured by 25 % and hence the claim would be of 75 % only. 149. A property is worth Rs 10 lakh and an individual insures it for a sum of Rs 12 lakh. In this case when a fire destroys the full property then the insurance company will pay the insured a sum of . Rs 500000 . Rs 800000 . Rs 1000000 . Rs 1200000 Solution: c Insurance is for indemnification purpose and there cannot be any profit from it. 150. There is an asset worth Rs 10 lakh and this is insured for a sum of Rs 8 lakh. There is a loss wherein half of the asset is destroyed. The insurance company will pay a sum of d. Rs 320000 d. Rs 400000 d. Rs 500000 d. Rs 800000 Solution: b 151. There is an excess of 10% in a policy and in the example where there is an amount of Rs 20000 that will be the claim then the insurance company will pay d. Rs 0 d. Rs 2000 d. Rs 18000 d. Rs 20000 Solution: c Excess is a compulsory deductible from claims. Here 10 % and hence 2000 would be deducted. 152. There is an excess of Rs. 5000 in a policy and the amount spent for the expense that has to be reimbursed under the policy comes to Rs 4500. The insurance company will pay d. Rs 0 d. Rs 500 d. Rs 4500 d. Rs 5000 Solution: a The purpose of excess is to keep away the small claims and reduce the expenses of servicing claims. So, nothing would be paid here as excess is 5000 and loss is 4500 which is below excess limit. 153. A policy has an excess of Rs 5000 and a limit of Rs 1 lakh. When there is a claim of Rs 55000 by the person insured then the insurance company will pay a sum of d. Rs 0

d. Rs 5000 d. Rs 50000 d. Rs 55000 Solution: c 154. A policy has a limit of Rs 1 lakh. When there is a claim of Rs 200000 by the person insured then the insurance company will pay a sum of d. Rs 0 d. Rs 50000 d. Rs 100000 d. Rs 200000 Solution: c The claim cannot be more than the limit fixed in the policy. 155. There is an asset worth Rs 10 lakh. A person insures this for Rs 5 lakh each with 2 different insurance companies. If the entire asset is destroyed then each insurance company will pay d. Rs 250000 d. Rs 500000 d. Rs 750000 d. Rs 1000000 Solution: b Both the companies would contribute to the loss. 156. There is an asset worth Rs 20 lakh. A person insures this for Rs 20 lakh each with two insurance companies. If this is destroyed to the extent of 50% then the amount that will be paid by the insurance companies together is d. Rs 1000000 d. Rs 2000000 d. Rs 2500000 d. Rs 4000000 Solution: a In insurance there is no scope for profit or gain, only indemnification. 157. A person takes insurance for Rs 5 lakh from one insurance company and Rs 3 lakh from another for the asset worth Rs 8 lakh. In this case when there is a loss of Rs 6 lakh then the total sum paid by the two insurance companies put together will be d. Rs 5 lakh d. Rs 6 lakh d. Rs 8 lakh d. None of the above Solution: b Both the companies would contribute to the loss. 158. If the rate of insurance is Rs 3 per thousand per year and there is an insurance policy of Rs 3 lakh then the premium will come to d. Rs 3000 d. Rs 6000 d. Rs 9000 d. Rs 9900 Solution: c 159. The following figure is not a premium figure paid by a holder of an insurance policy d. Rs 2500 d. Rs 1345 d. Rs 3546 d. Rs 1.2 per thousand Solution: d Rate is the price per unit of coverage i e 1000 in insurance while premium is the total cost or price paid for the coverage. 160. An insurance company has been in the business of insurance in particular country since the last 15 years. The company actively uses the data that has been generated for the past several years for the purpose of developing the insurance rates that it will charge its customers. This process is known as d. Insurance coverage d. Loading d. Premium process d. None of the above Solution: b 161. Premium on Motor Insurance policy does not depend on which one of the following factors? d. Zone of operation of the vehicle d. Insured Declared Value (IDV) of the vehicle d. Cubic capacity of the vehicle d. Age of the owner of the vehicle Solution: d 162. In several unit linked polices the _____ charge can be as high as 30-35% d. First year charge d. Administrative charge d. Risk charges d. Asset management fees Solution: a

Solution: a Premium allocation charges are generally higher in the 1 st year in ULIP plans. 163. A person took a life insurance policy on 2 October 2005. 25 days later he slashed his wrists after an argument with his family and consequently died. The claim under the policy a. Will be paid because the policy is in his name a. Will be paid because the policy has completed 15 days a. Will not be paid because he did not die due to a disease a. Will not be paid because of coverage under the suicide clause Solution: D 164. A person has a premium due on 15 November. Since he was traveling abroad he has not been able to pay the premium. After he is back on 25 November can he pay the premium d. Yes because a person can pay premium anytime d. Yes because the time falls within the grace period clause of the policy d. No because the due date was the last date for payment d. No because the premium can only be paid after a month Solution: b 165. A person wants to ensure that while he has taken the necessary life insurance cover there is also some e protection to him and his family in case there is a serious illness that affects the person and which can drain him of financial resources. In such a situation one can take the option of which rider d. Waiver of premium d. Accidental death d. Critical illness d. Guaranteed insurability Solution: c 166. When a rider is taken on an insurance policy along with an additional benefit there would be an additional cost in terms of d. Higher premium d. Lower premium d. Lower benefit d. Same benefit Solution: a 167. A person has a health insurance policy. He had to be admitted to a hospital for the operation and after this was completed he paid the bill and forwarded the insurance claim to his insurance company. After making the necessary checks the insurance company paid the amount. This is an example of a d. Indemnity plan d. Managed care plan d. Disability insurance d. Accident insurance Solution: a 168. A person owns a large house, which is lent out to tourists as a hotel. During the rains the road leading to the property was washed away leading to loss of income as the tourists were not able to come to the property. This is an example of d. Direct property risk d. Indirect property risk d. Loss of income d. All of the above Solution: c 169. A person enters into a contract in the course of his printing business. Due to the non fulfillment of the contract he was required to pay damages to some other party. This is an example of d. Statutory liability d. Personal liability d. Professional liability d. All of the above Solution: c 170. The overflowing of the river led to floods in Surat this year. After being trapped in their houses for over 5 days the Soni family was able to have access to food and water. In order to tackle such a situation in the future the family moved to Jaipur. This action of the family tackling the risk will be classified as d. Risk transfer d. Risk avoidance d. Risk retention d. Risk reduction Solution: b 171. A person has a fire in his stock house and goods to the extent of Rs. 235000 are destroyed in the fire. There is no other loss because of the fire and the person after looking at the situation files a claim for Rs. 275000. This is an example of d. Normal claim d. Fraudulent claim d. Inflated claim d. None of the above Solution: c

172. A painting is insured with a private collector for Rs. 25lakh. He quietly ships off the painting to one of his associates and then claims the insurance with the company for the required sum stating that the painting was stolen while he was on vacation. This is an example of d. Normal claim d. Fraudulent claim d. Inflated claim d. None of the above Solution: b 173. A person gives out his motorcycle on lease to his friend with the condition that any tax or repairs that will arise on the vehicle will have to be borne by the user. In this case the user has an insurable interest because of d. There is no insurable interest d. Common law d. Contract d. Statue Solution: c 174. A person has borrowed Rs. 2 lakh from his friend. Out of this a sum of Rs 50000 has already been paid back. The amount of insurable interest of the person who had loaned the money in the life of the borrower is d. Rs 0 d. Rs 50000 d. Rs 150000 d. Rs 200000 Solution: c 175. A person has a share of 50% in a property with his partner. The property is valued at Rs. 20 lakh. If the person insures the full value of the property of Rs 20 lakh then he will be allowed the insurance for d. Rs 0 d. Rs 1000000 d. Rs 2000000 d. None of the above Solution: c 176. In the above case the person can be deemed to be an agent for the cover of d. Rs 0 d. Rs 1000000 d. Rs 1500000 d. Rs 2000000 Solution: b 177. A person Sharad own various properties in the city of Jaipur. He has an agent Jayesh who operates and undertakes several activities on his behalf. Jayesh goes to an insurance company to take insurance on the property on behalf of Sharad. In this case there d. Is no insurable interest d. It is upto the insurance company to decide on the matter d. There is an insurable interest for the agent on behalf of the principal d. None of the above Solution: c 178. A person got married in the last week of December 2005. In the month of April 2006 the husband wants to know whether he has insurable interest in the property of his wife d. No he has insurable interest only in his own property d. No there cannot be any question of insurable interest for a year after marriage d. Yes there is insurable interest because the marriage is not registered d. Yes because souses have insurable interest in each others property Solution: d 181. Vinayak, 36 years and married, works for a multinational firm, which provides adequate medical and related covers. He is also able to accumulate sick leave. He already has his own home and savings of Rs. 35 lakh, which are well invested. Which insurance cover does he require the most? d. Life Cover d. Medical Cover d. Property Insurance d. Temporary Total Disablement Cover Solution: a 182. __________ Insurance protects oneself against losses as a result of lawsuits d. Life d. Medical d. Professional Indemnity d. Home Solution: c 183. Loss of income of earning member of family is a ___________ type of risk. d. Liability d. Personal d. Property d. Professional Solution: b 184. Only a _________ insurance policy can be assigned d. Health

d. Life d. Property d. Liability Solution: b 185. General insurance policies shall be void unless the person affecting the insurance has an insurable interest in that property at the time the policy is affected as well as at the time of the occurrence of the insured event. d. True d. False Solution: a 186. If the life insurance policy is endorsed under MWP Act, 1874 then _________________. d. Creditors have first claim over policy proceeds d. Creditors have no claim over policy proceeds d. Creditors have residual claim over policy proceeds d. Income tax authorities have claim over policy proceeds Solution: b 187. Which of the following statements about law and the insurance agent is (are) true? i. An insurer is responsible for the acts of its agents when the agents are acting within the scope of the agency agreement. ii. Consumers should not assume that an agency relationship exists just because someone claims to represent an insurance company. d. I only d. II only d. Neither I nor II d. Both I and II Solution: d 188. Kavita purchased collision insurance on his new car. While Kavita was driving home from work, another driver failed to stop at a stop sign and hit Kavitas car. Kavita phoned his insurance agent and reported the accident. The agent said, Dont worry, Kavita, well pay to get your car fixed. And after we pay for the damage to your car, we will try to collect from the driver who damaged your car. The process the agent described is called d. Waiver. d. Consideration. d. Estoppels. d. Subrogation. Solution: d 189. When someone reasonably relies upon a representation of fact, what legal doctrine prevents the representation of fact from being retracted if the individual who relied upon the representation of fact would be harmed? d. Estoppels d. Warranty d. Waiver d. Agency Solution: a 190. The assignment of property insurance by the seller of the property to the purchaser of the property is only valid if the insurer approves the assignment. The reason that the insurer must approve the assignment of a property insurance policy is that d. Insurance contracts are contracts of utmost good faith. d. Insurance contracts are bilateral contracts. d. Insurance contracts are personal contracts. d. Insurance contracts are aleatory contracts. Solution: c 191. When Maria applied for a life insurance policy, she answered No in response to the question Have you visited a doctor for any reason during the previous 12 months? In fact, Maria visited a doctor five weeks ago after experiencing chest pains. She was referred to a specialist who determined that Maria has severe heart disease. If Maria dies shortly after the life insurance policy is issued, upon what grounds will the insurer be successful in denying the claim? d. Concealment d. Waiver d. Warranty d. Misrepresentation Solution: d 192. Which of the following statements is (are) true with respect to the human life value approach? i. The human life value approach considers all sources of income that the family receives. ii. The human life value approach does not consider the time value of money--future cash flows are not discounted back to present value. d. II only d. Neither I nor II d. I only d. Both I and II Solution: b

193. An individual who is injured as result of a tort may bring a legal action against the person who caused the injury. In a court of law, the injured party is known as the d. Tortfeasor. d. Plaintiff. d. Attorney. d. Defendant. Solution: b 194. In some situations, it is not necessary to prove fault for injuries resulting from an act for the act to be deemed negligent. Such situations are described as involving d. Comparative negligence. d. Contributory negligence. d. Breach of contract. d. Strict liability. Solution: d 195. Conditions that increase either the frequency or severity of loss are called: . Subrogation. . Risks. . Hazards. Solution: c 196. What is the main responsibility of the underwriting department of a life insurance company? . To guard against adverse selection. . To set a limit on the amount of insurance issued. . To set adequate insurance rates. . To avoid exposures that could result in loss. Solution: a The underwriter tries to manage the risk of adverse selection. C is incorrect, because the rate is usually set after establishing underwriting standards. D is incorrect, because the purpose is not to avoid exposure but to manage risks. B is a responsibility of underwriters but is not their main responsibility. 197. Liability arising out of work done by independent contractors is known as d. Contingent liability. d. Completed operations liability. d. Premises liability. d. Contractual liability. Solution: a 198. Which of the following statements is (are) true with respect to Workers Compensation and Employers Liability Insurance? I. Workers compensation insurance is characterized by deductibles and coinsurance. II. Workers compensation provides coverage for occupational injury and disease. d. I only d. II only d. Neither I nor II d. Both I and II Solution: b 199. An individual may commit an act that results in bodily injury or damage to someone's property. A court of law may order the person responsible for the wrongful act to pay damages to the party who was injured. This type of risk is called the d. Speculative risk. d. Property risk. d. Liability risk. d. Fundamental risk. Solution: c 200. Two ways of assessing life insurance needs is a need based approach and the other is the income replacement method. What in your judgement would be the life cover required for Mr.Rao on the basis of each of the two approaches. Mr. Rao is the sole income earner in the family. Mrs. Rao is a homemaker. They are aged 40 and 36 respectively. Life expectancy for both of them is another 40 years. They have no children. Other information you have is: Current investment port folio - Rs.20 lakh.; Estimated final Expenses - Rs. 1 lakh.; Present annual expenses-Rs. 4 lakh (including a lakh of Mr. Rao's personal expenses); Mr. Raos post-tax income in hand-Rs.3.5 lakh.; Assume a post tax, post inflation return/discounting factor of 3%.. Calculate the insurance requirement under the Needs Based method. d. Rs.14.2 lakhs d. Rs.14.8 lakhs d. Rs.15.80 lakhs d. Rs.15.4 lakhs Solution: d Family.s expenses after death of Mr.Rao will fall to Rs.3 lakhs. Find Future value of these expenses for 40 years with inflation at 4%. Hence Future value is Rs.14.40 lakhs. Add Rs.1 lakh as Final expenses. Hence Insurance requirement will be Rs.15.40 lakhs. Use the information below to provide answers to Questions 201, 202 and 203 Your client earns Rs. 60,000 annually. She is married and has a child aged three.

If she should die, she wants her family to receive Rs. 45,000 annually for the next 20 years, with the first payment due on her death, and to provide Rs. 120,000 for her childs college education in fifteen years time. She also wants to set up a final expense fund of Rs. 15,000, and pay off the mortgage, automobile loan, and outstanding credit card balances. Extracts of her personal balance sheet are as follows: Assets Condominium Rs. 210,000 Personal Property 50,000 Automobile 40,000 Mutual Funds 50,000 Bank Deposits 10,000 Life Insurance 100,000 Mortgage Reducing Term Assurance 100,000 Group Death Benefit 30,000 Total 590,000 Liabilities Automobile Loan Rs. 15,000 Mortgage 100,000 Credit Card Balance 5,000 Total 120,000 Assuming that the life insurance proceeds and liquid funds can earn 6% annually, calculate 201. The present value of the annual incomes b. 500,246 b. 516,146 b. 547,115 b. 900,000 Solution: c Pmt = 45000 ; nper = 20 years ; rate = 6 % pv= 5,47,115. 202. The present value of her childs college education cost d. 50,071 d. 55,921 d. 119,827 d. 130,875 Solution: a fv= 120000 ; nper = 15; rate = 6 % pv= 50071. 203. The minimum amount of additional life insurance your client should purchase to attain her financial goals. d. No insurance is needed d. 143,000 d. 152,000 d. 440,000 Solution: b (Total Liabilities = 597,186 + 135,000 Total Assets = 590,000) 204. Calculate half yearly premium on the basis of the following data: Plan Term endowment 25 years, tabular premium Rs.53.40 per thousand Accepted with DAB extra Re.1 per thousand S.A. Occupation Extra Rs.3 per thousand S.A. Rs.1, 50,000 Rebate Rs.2 Yearly Mode = Rebate less 3% Half yearly premium less 1.5% Quarterly premium Nil Monthly premium plus 5% d. 8070 d. 4095 d. 7740 d. 8610 Solution: b Tabular premium payment mode discount large SA discount +occupation + DAB 53.40 0.80 = 52.60 2.00 = 50 .60 + 3 + 1 = 54.60 54.60 * 150 = 8190 /2 = 4095 205. Calculate loan amount: Sum Assured Rs. 1, 50,000 Mode of Payment Half yearly Rebate for half-yearly mode 1% Date of commencement 13.02.88 Date of last premium paid 13.02.92 Date of calculation 18.03.95 Table Term 16-50

Surrender value Rs. 52.3% Vested bonus declared from 3/88 to 3/91: Rs.150 per thousand sum assured Interim bonus declared on 31.03.92: Rs.75 per thousand sum assured. Loan: 90% of surrender value if the policy is at force or 85% of surrender value if policy is in paid up condition. a. 18,828 b. 16,004 c. 24,711 d. 21004 Solution: b Paid-up value No. of premiums paid X sum assured / No. of premium payable + bonus if any. Surrender value paid up value X S.V factor/100 Paid-up value = 9 X 1, 50,000/100 = Rs.13, 500 + 22, 500 = 36, 000 (As last premium paid was 13.02.92, the interim bonus on 31.03.92 is not taken for calculation loan amount) Surrender value = 36,000 X 52.3/100 = Rs.18, 828 Loan amount: 85% of surrender value, as it is paid-up condition 18, 828 X 85/100 = Rs.16, 003.8 or 16, 004 Paid up value = 9 * 150000 / 100 + 22500 = 36000 SV = 52.3 / 100 * 36000 = 18,828 Loan 18828 * 85 / 100 = 16004. 85 % because policy is though paid up but lapsed. 206. Calculate paid-up value: Sum Assured Rs.1, 00,000 Plan Without Profit Endowment Term 20 years Date of commencement 20.02.1995 Last premium paid 20.02.2000 Quarterly Mode Date of Birth 20.02.1950 Tabular premium 50.05 a. 25,000 b. 50,000 c. 26,250 d. 30,250 Solution: c No. of premium paid X sum assured/number of premium payable Rs.1, 00,000 X 21/80 = 26, 250 Paid-up value Rs.26, 250 Use the information provided below to provide answers to Questions 208 and 209 The sales literature of an insurer, pertaining to a particular participating policy, claims that in the past, the company had paid maturity benefits of two-and-a half times the basic sum assured on their 15-year endowment policies. The company uses the uniform compound reversionary bonus system and the terminal bonus system to distribute profits to its participating policyholders. Assuming a uniform reversionary bonus rate of 35 per thousand sum assured was declared over the last 15 years, a basic sum assured of 50,000 and level annual premiums of 5,066. 207. Ignoring mortality, calculate the terminal bonus content in the maturity proceeds d. 50,000 d. 41,232 d. 83,767 d. 75,000 Solution: b 208. Ignoring mortality, calculate the internal rate of return implicit in the maturity proceeds. d. 5% d. 6% d. 7% d. 8% Solution: b find rate given Nper = 15 ; pmt = 5066 ; fv = 125000 ; rate = 6 % 209. Which of the following is needed to calculate the client's human life value? i. Average annual earnings to the age of retirement. ii. Selection of an appropriate capitalization rate iii. Costs of self-maintenance. iv. Number of years from the client's present age to the contemplated age of retirement. d. I, ii

d. I, ii, iii d. I, iii, iv d. I,ii, iii,iv Solution: d The question asks what is needed to calculate the value of a life. The human life value is determined by finding the present value of the future cash flows (the clients annual salary). A, B, C,anf D are all needed in this calculation. 210. When fine arts or antiques are insured under a homeowners policy by an endorsement, d. Coverage is usually on a replacement cost basis. d. Coverage is usually on an actual cash value basis. d. Coverage is usually provided on a valued basis. d. The perils are the same as the homeowners policy to which the endorsement is attached. d. Coverage limits are the same as the homeowners policy to which the endorsement is attached. Solution: c 211. Which of the following is (are) true regarding the ownership of life insurance? A policy can only be issued to the insured. Generally, assigning a policy requires proof that the insured is still "insurable" meaning still in good health. Only a person with an insurable interest, generally a relative, a business associate, or lender, can be named as a beneficiary. The owner can assign (transfer) the policy to whomever he or she chooses, even if the assignee has no insurable interest. d. 1, 2, and 3 only. d. 1 only. d. 2 and 4 only. d. 4 only. Solution: d Life insurance policies can be issued to anyone with an insurable interest. The policy may be assigned to anyone and anyone may be named as beneficiary. The insured need not be insurable at the time of assignment 212. The following are statements made concerning contracts of insurance. Identify the statement/s that is/are correct. I. Property and liability insurance contracts are freely assignable by Policy owners without the insurers prior approval. II. The requirement that the insured must cooperate with the insurer in defending a liability claim is an example of a condition precedent. a. I only b. II only c. I and II d. Neither I nor II Solution: d 213. A son cannot insure his parents life. a. True b. False c. True only if both parents are earning a regular income. d. True only if the son is dependent on the parents. Solution: a 214. _________ insurance policy cannot be assigned. a. Term b. Endowment c. Money back d. ULIP Solution: a 215. An assigned life insurance policy can revert to the original policy holder once the liability is paid off. a. True b. False c. It can revert only at the discretion of the assignee. d. True, but the original policy holder will not receive any of the interim bonuses which e. have been declared on the policy. Solution: a 216. The minimum capital required for an insurance company as per the IRDA guidelines is Rs. _______ crores. a. 50 b. 75 c. 100 d. 200 Solution: c 217. Maximum sum assured available under a Life insurance policy currently is Rs. ______ lakhs a. 50 b. 75

c. 100 d. No ceiling is mandated Solution: d 218. TPAs are being employed currently in _________ policies. a. Medical Insurance b. Motor Insurance c. Life d. Both a and b Solution: d 219. Mrs. Mishra is a female assessee aged 55. The maximum amount of income tax deduction that she will get on account of payment of premium on a Medical Insurance policy is Rs. _____ a. 10000 b. 12500 c. 15000 d. 20000 Solution: a 220. The Insurance Act was passed in _________ a. 1956 d. 1938 d. 1872 d. 1881 Solution: b 221. Clause pertaining to Contribution and Subrogation are contained in ______ insurance contracts. d. Life d. Medical d. Accident d. Pension Plan Solution: b 222. The cost of insurance premium is ________ proportional to the odds of an event occurring. d. Directly d. Inversely d. Indirectly d. Data insufficient Solution: a 223. Wearing a helmet while riding a bike is an example of ___________ d. Risk Retention d. Risk Reduction d. Risk Sharing d. A combination of all the three options Solution: b 224. A start-up coming approaching the stock market with an IPO is an example of d. Risk Retention d. Risk Reduction d. Risk Sharing d. A combination of all the three options Solution: c 225. Mr. Vinod has purchased medical insurance policies from two companies viz. X and Y for a sum of Rs.5 lakhs each. He files for a claim of Rs.5 lakhs from X, who reimburses him for the same. X then approaches Y and recovers half this amount from it. X has employed the principle of _______ d. Contribution. d. Subrogation d. Collaboration d. Intimidation Solution: a 226. The following are examples of ___________. I. Outstanding balance on credit card II. Short-term loans outstanding III. Bank overdraft e. Long term liabilities e. Current liabilities e. Long term assets e. Current Assets Solution: b 227. In India, life insurance has been traditionally sold more as a __________ product. d. Protection d. Investment d. Tax-saving d. None of the above Solution: c 228. The rights of a beneficiary under an MWP life insurance policy are not affected by divorce. d. True d. False d. False in the case of Hindus d. True only in the case of Muslims Solution: a 229. A general insurance will usually insist that ________ is professionally valued before providing a Homeowners Insurance Cover.

a. Residence a. Electronic items a. Jewellery a. Furniture Solution: c 230. No claim bonus is not available in a ____________ policy d. Motor Insurance d. Health Insurance d. Home Contents d. Both a and b. Solution: c 231. Income, Income plus expenses, Multiple of salary are all methods of calculating insurance need by e. Research e. Rule of thumb e. Past record e. None of the above Solution: b 232. A client explains that she only wants an insurance policy that will cover her family against financial risk over the next five years, while she still has dependent children and a large mortgage. It is unlikely her income will increase over this period. What type of insurance is she looking for? d. An unit linked insurance plan d. Money back policy d. Term insurance with a level premium d. Term insurance with a stepped premium Solution: c 233. Which of the following is type of pecuniary insurance? . Commercial vehicle insurance . Engineering insurance . Money policy . Workman's compensation Solution: c 234. Participating policies are those where _____________. d. Both insurer and insured participates in each others loss d. Insured participates in running of insurance of company d. Insured participates in surplus of insurance company d. Insurer participates in loss of insured Solution: c 235. There is an excess of Rs 5000 for a health insurance policy. In a case where there is an expense of Rs 50000 for the insured then the insurance company will pay d. Rs 5000 d. Rs 45000 d. Rs 50000 d. Rs 75000 Solution: b Excess is a compulsory deductible and hence 45000 would be paid. 236. Anand was driving his car home from work, when a pit dug by the municipal corporation in the road, remained open and unmarked. He met with an accident and had to be hospitalised for 3 months. What are the insurance claims that can a rise from this accident? d. Anand can claim personal insurance for the accident, as it was not caused by negligence on his part; the municipal corporation cannot claim third party loss insurance to pay damages to Anand, as it was negligent. Anand can claim insurance for damage due. d. Anand can claim temporary disability insurance and insurance for his damaged car d. Anand has to apply to the municipal corporation for damages, which the corporation will pay out of its claims for liability to third party. His motor insurance will cover damages to his car. d. Since the municipal corporation was negligent, it would not be able to lodge a claim to recover payment of damages to Anand. Anand will only receive motor insurance claims on his car. Solution: a 237. Sujata was standing on the terrace of her building hanging out clothes. She accidentally fell off and landed on the sunshield of the next floor, which crashed and damaged the car of her neighbour parked below. What are the insurance claims that arise from this event? d. Sujata can claim personal accident insurance. Both her neighbours will claim property insurance for the freak accident. d. Sujata cannot claim accident insurance as the accident was caused by her negligence. Her neighbours can claim property insurance cover for loss to their property d. Sujata's neighbours will collect damages from her, which Sujata can pay out of insurance cover for losses to third parties. d. Sujata's neighbours will not be able to claim insurance, as the damage to their property due to such freak accidents is not usually covered by insurance. Sujata will be able to claim her accident insurance, as she did not fall intentionally. Solution: a

238. During retirement, a person would maintain a high level of life insurance protection to: (1) Pay estate duty (2) Provide succession or dependent income (3) Repay outstanding debt (4) Build cash value . (1) and (2) . (1) and (3) . (1), (2) and (3) . None of the above Solution: b 239. Your client has bought life insurance and medical insurance, but has not bought a cover for permanent disability. His argument is that he is paying too much by way of premium for risks that he believes are "far-fetched" and "not likely to affect him". What would you advise the client? d. A financial planner can persuade the client to consider the losses from permanent disability and highlight the risks to the client and recommend an appropriate policy for him. d. If a client is not willing to bear the costs of premium, it can be assumed that he is willing to bear the costs of risk retention. Insurance may not be necessary in such cases. d. If losses that would occur to the client in the event of permanent disability are higher than what he can bear, the client is better off buying insurance. The costs of insuring against losses, which have lower probability of happening, will in any d. The amount of insurance a person will buy depends on his perception of risks and their impact on him. It would not be possible to persuade this client to buy more insurance. Solution: a 240. Suresh has not bought accident insurance cover, though his two-wheeler is covered for damages from accidents. He wears a helmet and drives carefully. What can you say about his risk management? d. Suresh has insured the property risk. He controls some of his personal risk and retains the rest of the risk. d. Suresh has controlled his personal risk and insured his property risk d. Suresh has not done anything to manage his risks and has to immediately go for accident and personal risk cover. He cannot rely on third party damages alone to cover the risk of the road. d. Suresh has transferred his personal risk to other drivers of the road, insured his property risk and can claim damages is accidents are caused by third party negligence. Solution: a 241. Mrs. Rangnekar, a 40-year-old widow, has a 8-year-old son. Her current savings are not adequate to provide for her son's post graduate studies however she will be able to save it up by the time he finishes graduation i.e. when he is 20 years old. Mortality tables indicate that her life expectancy is another 30 years. Which of the following is true? d. She needs to insure her life for 12 years d. She does not need to insure her life d. She needs to insure her life for 30 years d. She needs to insure her son.s life for 30 years Solution: a 242. Holistic Institute of Wealth Management has a great Education centre and a full business. Dr.Shah, Head Academics is held in high esteem nationwide for his expertise, knowledge and lecture delivery. He has made a outstanding contribution for the last 8 years to enable market leadership for Holistic Institute of Wealth Management. The most appropriate policy that the owners may take in the interests of business on the life of Dr. Shah would be _____________. d. Personal Accidental Insurance d. Term Insurance d. Key Man Insurance Solution: c 243. The gross income of a person is at Rs 3 lakh when the average tax rate is 20%. Due to the receipt of an additional sum of Rs 2 lakh the average rate goes up to 25%. If there is then an additional sum of Rs 50000 spent for own expenses then the amount remaining for the family is d. Rs 325000 d. Rs 375000 d. Rs 425000 d. Rs 450000 Solution: a 244. Personal Data of Nilesh: Earnings: Rs. 2 lakh p.a Debts: Rs. 1,00,000 Total Expenses: Rs. 1,28,572 To support dependents 75% of pre-death salary (if no debt)

To support dependants spouse and one child, 75% of pre-death salary required (if no debt). Assuming simple interest of 7% p.a, As a Financial Planner find out the Insurance required by Capital Retention Approach and the times it will cover his current annual salary? d. Rs. 22,42,000 and 10 times of current annual salary d. Rs. 22,42,000 and 12 times of current annual salary d. Rs. 21,42,000 and 11 times of current annual salary d. Rs. 21,42,000 and 12 times of current annual salary d. Rs. 8,65,300 and 5 times of current annual salary d. None of the above Solution: b 245. Brijesh has a health insurance policy with sum assured Rs.10 Lakhs. The deductible is Rs. 2,500. Brijesh had a operation to remove his appendix which cost him Rs. 30,750. He also lost Rs. 5,000 in wages as he could not work for 2 weeks. What would be the amount paid by the insurance company? d. Rs. 30,750 d. Rs. 28,250 d. Rs. 33,250 d. Rs. 35,750 Solution: c 246. Normal medical policies do not have a loss of earning cover. Which one of the following does not come under principal of merit rating? d. Schedule rating d. Experience rating d. Retrospective rating d. Judgement rating Solution: d Rating could be- IV. Individual or judgmental IV. Class or manual IV. Merit rating Schedule, Experience & Retrospective 247. Bima Kavach Yojana is a policy designed for which area a. Urban Area b. Rural Area c. Both of them d. None of them Solution: b Bima kavach Yojna is a premium back term plan by Birla Sunlife specially for rural masses. 248. Which one of the following is not the Money Back Policy of LIC d. Jeevan Chhaya d. Jeevan Anand d. Jeevan Surbhi d. Jeevan Rekha Solution:b Jeevan Anand is combination of whole life and endowment, While all other 3 are money back policies. All these policies are from LIC. 249. Which one among the following professionals are not eligible for Professional Indemnity Cover? a. Plastic Surgeons a. General Physician a. Dentists a. Consulting Physician Solution: a 250. "Rajarajeswari Mahila Kalyan" is a policy designed for women, it comes under which category of Insurance: d. Health Plan d. Money Back d. Term Plan d. Personal Accident Solution:d Rajarajeshwari mahila kalyan is a Disablity policy by United India Insurance Company. 251. In Mediclaim, what is the limit of premium which is totally exempt from Income Tax? d. Rs.10,000 , if both the partners are covered d. Rs.10,000, if paid by a cheque d. Rs.15,000, for family package d. Rs. 20,000, if insurer has life insurance policy 252. A person is looking at evaluating the life insurance needs that will arise for him. The estimate is that final medical expenses will be around Rs 1.5 lakh, the funeral charges Rs 20000. In addition his wife will require Rs 2 lakh in her retirement period. In this case his immediate cash needs are d. Rs 170000 d. Rs 370000 d. Rs 220000 d. Rs 350000 Solution: a 150000 + 20000 = 170000.Because 2 lakhs is income need.

253. In the above case, If the Readjustment Period Expenses at todays value is Rs 3 lakhs and the Dependency Period Expenses at todays value is Rs 12 lakhs, plus the Blackout Period Expenses are Rs. 5 Lakhs , As a Financial Planner find out his Net Income Needs. d. Rs 25.70 lakhs d. Rs 15 lakhs d. Rs 20 lakhs d. Rs. 22 lakhs d. None of the above Solution: d 254. In the above case, as a Financial Planner find out his Insurance Requirement as per Needs Approach, Note his Investments in Mutual Funds and Stocks are valued currently at Rs. 8 Lakhs as well as he has an exisiting Insurance Policy with a Sum Assured of Rs.2.70 Lakhs. a. Rs. 25.70 Lakhs a. Rs. 17.70 Lakhs a. Rs. 15 Lakhs a. Rs. 13 Lakhs a. None of the above Solution: d 255. A person lives in a nuclear family consisting of his wife and two children. He wants to assume his premium needs at 5 per cent for self plus 1 percent for other members of his family. He has an income of Rs 5 lakh and an expense figure of Rs 3 lakh during the year. The expense is expected to go upto Rs 3.25 lakh next year. The amount of insurance cover required here is d. Rs 5 lakh d. Rs 10 lakh d. Rs 15 lakh d. Cannot be determined Solution: d Only premium payable can be worked from the information and not the cover. 256. A person lives in a nuclear family consisting of his wife and two children. He wants to assume his premium needs at 5 per cent for self plus 1 percent for other members of his family. He has an income of Rs 5 lakh and an expense figure of Rs 3 lakh during the year. The expense is expected to go upto Rs 3.25 lakh next year. The amount of premium that will be used for the insurance cover will be d. Rs 40000 d. Rs 45000 d. Rs 50000 d. Cannot be determined Solution: a 5 + 1 + 2 = 8 % of income.8 / 100 * 500000 = 40000 257. A person lives in a nuclear family consisting of his wife and two children. He wants to assume his premium needs at 5 per cent for self plus 1 percent for other members of his family. He has an income of Rs 5 lakh and an expense figure of Rs 3 lakh during the year. The expense is expected to go upto Rs 3.25 lakh next year. As a Financial Planner find out the amount of premium that will be used for the insurance cover by employing Premium as a Percentage of Income Method. d. Rs 40000 d. Rs.41000 d. Rs 16000 d. Rs 14000 d. None of the above Solution: a 258. Refer to the above case study, As a Financial Planner find out the Total Insurance cover he can avail by using the Premium as a Percentage of Income Method (Note Insurance Premium for age 36 is Rs. 4.00 per Rs. 1000 sum assured. c. Rs 35 lakhs c. Rs 100 lakhs c. Rs 40 lakhs c. Rs. 102.50 lakhs c. None of the Above Solution: b 4 / 1000 so, for 40,000 one can buy for 1 crore. 260. In case of a need that will amount to Rs 3 lakh for 5 years after a period of 3 years and where the rate prevailing is 5 % then this figure converted to an insurance need today comes to d. Rs 1121989 d. Rs 1298843 d. Rs 1356474 d. Rs 1456786 Solution: a 261. Which of the following statements is (are) true with respect to pure risks? i. Pure risks may produce either a profit or a loss. ii. Premature death and damage to property caused by a fire are pure risks. f. I only

f. Both I and II f. Neither II nor I f. II only Solution: d 262. The modern portfolio theory suggests that the portfolio returns can be optimized by _______ d. Investing in diversified equity funds. d. Investing in treasury bills and equities. d. Laddering the bond portfolio. d. Moving closer to the efficient frontier in terms of the risk return equation. 263. Karan wants to withdraw Rs. 1200/- at the end of each month for the next 5 years. He expects to earn 10% interest compounded monthly on his investments. What lump sum should he deposit now? d. Rs. 56949 d. Rs. 58630 d. Rs. 56478 d. Rs. 59119 Solution:c find pv given rate 10 % /12 ; nper = 12 * 5 ; pmt = 1200 pv= 56478. 264. From the following particulars, calculate paid-up value: Sum assured: Rs.50,000, Plan: Endowment without profit Term: 20 years Date of commencement: 19/02/1994 Last premium paid yearly: 14/02/2000 a. Rs. 50,000 b. Rs. 17,500 c. Rs. 15,000 d. Rs. 18,000 Solution: b 50000 *7 / 20 = 17500. 265. Case 1: The owner of a building with running machinery seeks to insure the property by choosing a certain sum assured, as full value of the risk to be covered. Case 2: the owner of a property suffered a fire loss. He seeks to restore the cover value to its original level to make up for the loss and is willing to pay additional premium for this purpose. (A) The former is a case of reinstatement as a basis of indemnity, while the second instance is one of reinstatement of sum insured following a loss under the policy. (B) No, it is in reverse order. c. Statement A is right c. Statement B is right c. Both statements A & B are wrong c. None of the above Solution: a 266. Ashish purchased a sofa set for Rs.50,000 and insured it on an actual cash value basis. Three years later the sofa set was destroyed in a fire. At the time of the loss, the furniture had depreciated to 50% of its value. The replacement cost of a new sofa set at the time of the loss was Rs.60,000. The deductible was Rs.3,000. What amount would Ashish get from the insurance company? d. Rs. 47,000 d. Rs. 57,000 d. Rs. 27,000 d. Rs. 22,000 Solution: d 50,000 25,000 [dep] = 25,000 . Claim to be paid= 25,000 3000 [ ded] = 22000. 267. There are 2000 buildings in a city valued at Rs. 20,00,000 each. All the buildings are insured upto 50% of their value. The following fire losses occur: 2 total losses 30 partial losses at Rs. 2,00,000 each. What would be the pure premium rate? a. Rs. 1.25 per thousand b. Rs. 2.00 per thousand c. Rs. 2.50 per thousand d. Rs. 4.00 per thousand Solution: c Total loss = 2 * 20 lakhs + 30 * 2 lakhs = 100 lakhs They are underinsured by 50 % so 50 lakhs / 2000=2500 / 1000 Rate = 2.50 268. Nilesh owns a laptop that was stolen. The laptop cost Rs. 35,000 when it was purchased one year back. A similar laptop can be bought today for Rs. 20,000. Assuming that the laptop was 50% depreciated, what is the amount payable by the insurer? Assume deductible to be Rs. 1,000. d. Rs. 9,000

d. Rs. 10,000 d. Rs. 19,000 d. Rs. 34,000 Solution: a 20000 10000 = 10000 1000 = 9000. 269. The net cost of insurance per Rs.1,000 based on the traditional net cost method will be: d. Rs. (-0.264) d. Rs. 1.499 d. Re. 0.517 d. Rs. 5.284 Solution: aTraditional net cost method= premium paid-[dividends+SV] --------------------------------------No. of years 45600 50884 / 20 = -264.2 / 1000 = -0.2642 270. The net cost of insurance per Rs.1,000 based on the surrender cost method will be d. Rs. (-0.264) d. Rs. (-0.404) d. Rs. 0.561 d. Rs. 1.577 Solution: c Surrender cost method = 79159 [ 24400 + 35260] / 34.719 271. The net cost of insurance per Rs. 1,000 based on the net payment cost method will be: d. Rs. (-0.264) d. Rs. 1.499 d. Rs. 1.823 d. Rs. 1.577 Solution: d Net payment method = 79150 24400 / 34.719 272. The net cost of insurance per Rs.1,000 based on the traditional net cost method will be: e. Rs. (-0.870) e. Rs. (-0.780) e. Rs. 20.790 e. Rs. 5.284 Solution: Traditional net cost method= premium paid-[dividends+SV] --------------------------------------No. of years 273. The net cost of insurance per Rs.1,000 based on the surrender cost method will be: e. Rs. (-0.780) e. Rs. 9.084 e. Rs. 21.510 e. Rs. 1.454 Solution: Surrender cost method = 79159 [ 24400 + 35260] / 34.719 274. The net cost of insurance per Rs.1,000 based on the net payment cost method will be: d. Rs. (-0.780) d. Rs. 9.084 d. Rs. 21.510 d. Rs. 1.454 Solution: Net payment method = 79150 24400 / 34.719 275. Ramesh has a family medical insurance policy with sum assured of Rs. 1,00,000. The policy has a deductible of Rs. 2,000 that applies to each covered person. He and his family are injured in an accident and have to be hospitalized for treatment. The treatment expenses of each are: Ramesh: Rs. 2,500 His Wife: Rs. 10,000 His Daughter: Rs. 7,000 His Son: Rs. 500 d. Rs. 20,000 d. Rs. 10,000 d. Rs. 17,000 d. Rs. 12,000 276. Ashok, born in 1950, has a life expectancy at birth of 65 years. Sita, his wife, born in 1955 has a life expectancy at birth of 70 years. Assuming that the life expectancies have not changed, Ashok is planning to buy an annuity to be paid to him or his wife till anyone of them is alive. He should buy an annuity for ________ years. e. 10 yrs. e. 12 yrs e. 07 yrs

e. 17 yrs Solution: d 277. For dependent parents, the least important risk to be covered is e. Life insurance cover e. Hospitalization / Sickness Insurance e. Disability Insurance e. Long term Care cover Solution: a 278. Suresh, the driver, driving Rams car (Ram was traveling in the car) was involved in a collision accident against the boundary wall of Rahuls house and also injured Rahul, Rams neighbour. It was later ascertained that driver Suresh was under the influence of alcohol. One of the perils mentioned below will not be covered while setting the claim: . Repair charges to Rams car . Boundary wall repair charges of Rams house . Medical treatment charges of Ram . Medical treatment charges of Rahul Solution: a 279. For guaranteed renew ability of term insurance policies, which of the following are important factors d. Age of the insured d. Premium rates d. Risk status d. All of the above Solution: d 280. A father takes out a policy to meet his sons enhanced educational needs when the son reaches 18 years of age. Unfortunately after paying 5 years premium, the father dies. By what means can the policy be kept in force without payment of further premiums so that the intended benefits are retained d. Accident benefit premium d. Lien on policy d. Premium waiver facility d. None of the above Solution: c 281. For professionals with limited time span of income earning possibilities, the following plan of insurance would be ideal d. Whole life d. Term assurance d. Immediate annuity d. Limited payment endowment plan Solution: d 283. Under an Overseas Travel Insurance policy, the maximum cover for third party liability is d. US $ 1,00,000 d. US $ 2,00,000 d. US $ 5,00,000 d. None of the above Solution: a 284. Which of the following statement(s) is / are True? Statement (A): The existence of insurance may make people to exert less effort to control losses. Statement (B): It might also induce commitment of frauds against insured. d. Both A & B statements above are true d. Statement A is true d. Statement B is true d. Both A & B statements are not true Solution: a 285. A Term Insurance plan can be for d. Constant cover d. Decreasing cover d. Increasing cove d. All of the above Solution: d 286. The actuary is responsible for d. Product design d. Premium rates d. Paid up and surrender value d. All of the above Solution: d 287. Premium is the legal consideration paid by the d. Insurer d. Insured d. Agent d. Broker Solution: b 288. Rate of premium depends on the basis of d. Age d. Term d. Policy type

d. All of the above Solution: d 289. The amount required to meet the risk of death for a given age in a particular year is called d. Risk premium d. Net premium d. Gross premium d. None of the above Solution: a 290. The proposal form includes d. Personal details of the client d. Policy details d. Information about the insure d. Risk factors in insurance business Solution: a 291. The proposal form has to be signed by d. The proposer and witnessed by a third party d. The proposer and the agent d. The proposer and the person whose life is to be assured d. Both A and C Solution: c 292. The surplus generated from the investment division of an insurance company, which is distributed amongst the policyholders is called as d. Bonus d. Interest d. Maturity value d. None of the above Solution: a 293. Every policyholder is given the option to choose premium payment ___________ d. Mode d. Frequency d. Frequency d. Both A and B Solution: d 294. Term insurance provides ___________ benefit d. Death d. Survival d. No d. Maturity Solution: a 295. An owner of a large group of privately owned residential property units is seeking cover from a general insurance company for (i) fire and other associated perils (ii) depreciation in the value of property (iii) complete destruction of property from terrorist strikes for the next six months. The chances are that the insurance company will____ d. Offer cover for (i), (ii) and (iii) d. Offer cover for (i) and (iii) d. Offer cover for (i) only d. Offer cover for (i) and (ii) Solution: c 296. Which of these are not contracts of indemnity? d. Life insurance d. Fire insurance d. Homeowners insurance d. Baggage insurance Solution: a 297. Consider the two statements below (A) While assessing financial liabilities, it is customary to include credit card debt, hire purchase of household goods, lease etc. under short term liabilities. (B) Longer term liabilities include personal loans etc. d. Statement A alone is correct d. Statement B alone is correct d. Statements A & B are correct d. None of the above Solution: a 298. Consider the two statements below: (A) Past losses may be a precursor of further losses in all types of life and general insurance business. (B) Past losses may be a precursor of future losses in some general insurance cases, critical illness and personal accident insurance cases. d. Statement A alone is correct d. Statement B alone is correct d. Statements A & B are correct d. Statements A & B are incorrect Solution: b 299. Which of these is involved in Needs Analysis? d. Needs Explaining

d. Needs Sequencing d. Needs Prioritization d. None of the Above Solution: c 300. The prescribed training program for life insurance or a general insurance agency that an applicant has to undergo is for ________. . 50 hours . 150 hours . 80 hours . 100 hours Solution: d 301. A Broker is characterized by the fact that he isd. Attached to the insurance company d. Representative of a group of companies d. An independent intermediary d. Employed by the insurance company Solution: b 302. Which agency conducts the pre-recruitment test for a prospective agent? d. The Insurance Company d. Insurance Institute of India d. IRDA d. Actuarial Society of India Solution: b 303. The IRDA allows a cooling period of ______ for the holder of a Life Insurance policy. d. 10 days d. 15 days d. 30 days d. 07 days Solution: b 304. A Composite Broker may be defined asd. A general and life insurance broker d. One who carries on business as a broker and agent d. An insurance and reinsurance broker d. A general insurance agent Solution: b 305. Within how many days of accepting a proposal must an insurer handover the copy to the insured? d. 10 days d. 15 days d. 07 days d. 30 days Solution: d 306. Consider the two statements below: (A) An important step in risk control is to spread out the cost of risks evenly over a period of time. (B) The objective should be to minimize the cost of risk. Of the above two statements, what should be the correct order of objectives d. Both A & B are of equal importance d. The given order: first A then B is correct d. B should precede A d. None of the above Solution: a 307. Life expectancy has been on the rise in India since 1921 because of d. an improvement in dietary practices d. Safer birth practices d. Enhanced health care amenities d. All of the above Solution: d 308. Which of these will NOT be considered a dependant while taking insurance? d. Children d. Relatives d. A spouse or partner d. None of these Solution: d 309. One must look at various stages of an individuals life to analyze his/her insurance need. These stages of life comprise d. Young family d. Young adult d. Matured couple d. All of these Solution: d 310. Which of these insurances are appropriate for a young adult? d. Household insurance d. Personal protection insurance d. Automobile insurance d. All of these Solution: d

311. Liability exposure is NOT d. Self - injury in an accident d. Injury to employees d. Bodily injury or property damage d. Professional negligence in giving advice Solution: a 312. Which of these is provided for under a Term Insurance policy? d. Only the premium is paid till the date of death d. Sum assured in case of death d. Sum assured plus bonus in case of death d. None of these Solution: b 313. A Participant Whole Life policy will allow the insurer to participate in d. The day to day running of the insurance company d. The board of the insurance company d. The profits of the company d. None of these Solution: c 314. Mr. Singh has applied for a bank loan of Rs. one crore to launch his own business venture. He is an Electronics Engineer by profession. The bank approves his loan with a conditional hypothecation of +25% and requests him to hypothecate the life policy as collateral security for mortgage redemption. Calculate the sum assured of the policy. d. 80 lakhs d. 75 lakhs d. 1.25 crore d. 2 crores Solution: c 315. __________ is a claim arising out of the life assured, surviving till the end of the selected term of the life insurance d. Annuity d. Maturity claim d. Death claim d. None of the above Solution: b 316. An Endowment policy may be defined as one that paysd. Premiums on death only d. Premiums paid on death or survival d. Assured benefits on death only d. Assured benefits on death or survival Solution: c 317. The disadvantage of insurance mechanism includes d. Reduction of uncertainty d. Operating expenses and moral hazard d. Capital gain and financial risk d. None of the above Solution: b 318. Which of these policies refer to a Home Loan/ Mortgage Protection? d. Credit insurance policy d. Property insurance policy d. Policy guaranteeing the mortgage loan d. None of the above Solution: c 319. Which of these is NOT provided for by an Educational Life policy? d. Continued Educational support to an insured individuals children, in case of his death d. Education to insured individuals up to a certain age d. Payment of educational expenses at a future time d. All of these Solution: b 320. What is a Cooling off provision in an insurance policy? d. When the insured individual can elect to cancel the policy and receive refund of the premium d. When the insured individual does not have to pay a premium d. When the insurance company cools off or stops its operations d. None of these Solution: a 321. If a claim arises out of the death of an insured individual, _____ would be paid the money. d. The first son of the insured individual d. The spouse of the insured individual d. The mother of the insured individual d. The person nominated under the policy by the individual, if any Solution: d 322. Insurance under Disability Insurance is not provided for___________ d. Disabilities caused by sickness

d. Disabilities caused by accidents d. Disabilities caused by hereditary causes d. None of these Solution: c 323. Under personal accident insurance, the amount insured is associated with_________ d. Income of the insured individual d. Family size of the insured individual d. Age of the insured individual d. Personal habits of the insured Solution: a 324. The insurance provided by critical illness covers d. Assured benefits for major illnesses d. Hospital expenses for major illnesses d. Long-term care after a major illness d. All of these Solution: a 325. Income Protection insurance allows for protection against loss of income in which of these instances? d. A business loss d. The insured individuals disability or sickness d. Unemployment of the insured individual d. Death of the insured individual Solution: b 326. A Hospital Expenses Insurance policy will NOT cover d. Expenses incurred due to outstation treatment d. Pre and post hospitalization expenses d. Hospitalization expenses following an accident injury d. None of these Solution: a 327. The initial procedure followed by a planner while reviewing a clients insurance program is d. To determine clients liability d. To analyze the insurance need of the client d. To clarify with the client if there are any changes in his needs. d. To establish the frequency of review of the clients planning process Solution: d 328. The evaluation of a clients program will conclude with___________ d. Executing the revised program d. Establishing a time for a future review meeting with the client d. Discussing with the client the need for the programs review d. Setting up a revised program after consulting with the client Solution: d 329. When there is the possibility of loss or no loss in an outcome, it is a case of d. Gambling d. Speculative risk d. Pure risk d. Special risk Solution: c 330. During the review process of the insurance plan, the planner must d. educate the client about the need for additional cover in case of changed circumstances d. let the client know if any new product will be more suitable for the client in place of the existing d. notify the client of the risks faced by him, which are not covered by his program d. all of these Solution: d 331. The _______ initiates the action with respect to maturity claims under a life insurance policy. d. planner d. policy holder d. intermediary who sold the policy d. insurance company Solution: b 332. The claim will be payable to _______ if the policy owner and the insured individual are different. d. the policy owner d. the spouse and children of the insured individual d. the legal heirs of the life assured d. the person nominated to receive the policy money Solution: d 333. Under a disability benefit policy, the waiting period is the period of time that must elapse d. before the issue of the policy document d. before the policy benefit will commence d. before the proposal can be considered d. before the settlement of claim after submission Solution: d 334. When should one inform the insurer in case of a health insurance claim? d. When the insured person is hospitalized d. When the health disorder is noticed

d. When the course of treatment for the insured person is complete d. When the bills of the hospital, where the insured individual was hospitalized, are paid. Solution: a 335. Which of these is applicable under property insurance? d. The amount payable is at the sole discretion of the employer d. The amount payable is guaranteed under the policy d. The amount payable is assessed according to the principle of indemnity d. None of these Solution: c 336. Which of claims is applicable under disability benefit claims? d. The policy holder has to undergo a medical examination by an approved medical examiner d. The policy holder has to submit medical certificates from the attending doctor d. The policy holder has to undergo specific medical examinations d. All of these Solution: d 337. The insurance planner or intermediary is responsible for_________. d. Health insurance claims d. Death claims in life insurance policies d. Disability claims in personal accident policies d. All of these Solution: d 338. _______ is the regulator for the insurance industry in India d. Controller of Insurance d. Insurance Regulatory and Development Authority d. General Insurance and Reinsurance Business d. Tariff advisory committee Solution: b 339. Consider the two statements below I. In life insurance, where the proposer was born would be deemed a material fact. II. In personal accident insurance, medical history would be deemed a material fact. d. Only statement I is wrong. d. Both statements I & II are correct d. Statement I is correct, while II is wrong. d. Both Statements I & II are incorrect. Solution: d 340. Which of these transactions are allowed by an Indian Insurance company? d. Life insurance and Reinsurance business d. Life and general insurance business d. General Insurance and reinsurance business d. Either life or general insurance or reinsurance business Solution: d 341. General insurance business comprises d. Fire insurance d. Marine insurance d. Miscellaneous insurance business d. All of these Solution: d 342. _______ transacted general Insurance business before the IRDA act was passed in 1999 d. General Insurance Corporation of India (G.I.C) d. General Insurance Corporation of India along with its 4 subsidiaries d. Life Insurance Corporation (L.I.C) d. All of these Solution: b 343. What is the minimum paid up capital required for establishing a new Life Insurance Company? d. Rs. 200 crore d. Rs. 100 crore d. Rs. 50 crore d. As fixed by the IRDA Solution: b 344. Consider the two statements below I. Insurance companies reduce risk based on law of large numbers. II. The increasing pool of resources accessible to an insurer reduces the risk of non payments. d. Statement II alone is correct d. Both Statements I & II are correct d. Both Statements I & II are incorrect d. Statement I alone is correct Solution: b 345. Mr. Mathew is the owner of a large group of privately owned residential property units seeking cover from a general insurance company for 1. Fire and other associated perils 2. Depreciation in the value of property 3. Complete destruction of property from terrorist strikes for the next six months

The chances are that the insurance company will d. Offer cover for 1, 2, and 3 d. Offer cover for 1 and 3 d. Offer cover for 1 d. Offer cover for 1 and 2 Solution: c 346. Ram and Raja are healthy, able bodied men working in mines. Ram lives on a flood plain and Raja lives on a hill next town. While talking out insurance, the insurance company proposes a higher premium for Ram. How would you discuss the present situation? d. Both are healthy, so both should be charged the same premiums d. They should get a discount for being healthy d. Living on a flood plain does not in any way affect the premium d. Ram should understand that that the risk is higher so the premium is also higher Solution: d 347. A property worth 10 lakhs has been preferred to have a cover of 6 lakhs by its owner. Claim has been logged for a loss of 2 lakhs. How much does the insurance company usually pay? d. Rs.2 lakhs d. Rs.6 lakhs d. Rs.1.2 lakhs d. Rs.1.5 lakhs Solution: c 348. Mr. Mani opted to have statutory minimum cover on his car to avoid higher premiums. While saving a child, his car hit a parked car causing damage to it of Rs. 5000 and then hit the wall causing further damage of Rs. 5000; his own car suffered damages worth Rs. 8000. How much claim is admissible? d. Rs. 18,000 d. Rs. 10,000 d. Rs. 5,000 d. None of the above Solution: b 349. Consider the two statements below: I Statements made by the proposer in the proposal form are representations. II All such statements are, by definition, material facts. d. Both statements are correct d. Statement II is incorrect d. Both statements I & II above are incorrect d. Statement II is correct Solution: a 350. The maximum rate of commission payable to Agents/ Advisors by the insurance companies for the first, second and the third year are d. 25%, 10% and 7% d. 15%, 15% and 10% d. 20%, 15% and 12.5% d. 35%, 7.5% and 5% Solution: d 351. The sum assured received under any policy is taxable if the premiums payable e) is more than 20% of the sum assured e) If it exceeds 30% e) If it exceeds 15% e) If it exceeds 12.5% Solution: a 352. Rearrange the sequence of the planners advises to the client for resolution of insurance complaints 1. Arbitration 2. Ombudsmen 3. In house complaint handling call of the insurer 4. Consumer Protection forums a) 1,2,3,4 b) 1,3,2,4 c) 1,4,2,3 d) 3,2,1,4 Solution: d 353. Following are the details of Mr. Alok are: Earnings Rs.1 lakh per annum Debts Rs.1,28,572 to support dependents 75% of pre-death salary (if no debt). Assuming simple interest of 7% per annum, how much per current annual salary he needs for life Insurance, if multiple method is used? a) 10 times b) 8 times c) 12 times d) 16 times

Solution: c 75000 * 100 / 7 = 1071429 + 128572=1200001 which is 12 times salary. 354. Mr. Sippy submitted a proposal on his own life on 1.3.2003. It was completed by the insurer and the Policy Bond was received by him on 8.3.2003. On 20.3.2003, he returned the Policy Bond stating the reason for his refusal to accept some of the terms printed in the Policy Bond and asked for refund of premiums paid. He is entitled to a) Any refund b) Half the amount c) 75 % of the amount d) No refund Solution: a because there is a 15 days cooling period where in one can ask for refund of premium by canceling the policy. 355. Identify the element mentioned below which not a factor for premium consideration in Overseas Medical/ Travel insurance a) Age of the proposer b) Purpose of the trip c) Duration of the trip d) Amount of life insurance held Solution: d 356. Tony buys an annuity from an insurance company, under which the insurer will pay him a certain sum periodically as long as he is alive. Billy buys an annuity from an insurance company under which the insurer will pay annuities to him( or his heirs) for a definite number of years, whether he is alive or not. 1. Tony has purchased a life annuity and Billy has purchased an annuity certain. 2. Tony has annuity certain and Billy has variable annuity. a) 1 is correct b) 2 is correct c) Both 1 and 2 are correct d) None is correct Solution: a 357. Manish and Sheela, a retired couple, opted for a joint life survivor annuity. Under this d) The annuity is payable as long as both of them are alive d) The annuity will cease on the death of the eldest of the annuities d) The annuity is payable as long as either one of them is alive d) The annuity payment will cease on the death of any one of the annuitants Solution: c 358. The least important risk, for dependent parents to be covered is d) Life Insurance cover d) Hospitalization/ Sickness Insurance d) Disability Insurance d) Long Term Care Cover Solution: a 359. Consider the two statements below: 1. Type of construction is considered for Fire Insurance 2. Driving track record is a material fact for Motor Insurance a) Both 1 and 2 are correct b) 1 is correct and 2 is incorrect c) 1 is incorrect and 2 is correct d) Both 1 and 2 are incorrect Solution: a 360. Consider the following two cases: After two years of taking out a life insurance policy, the assured dies after complications from emergency surgery after an acute attack of duodenal ulcer. Enquiries reveal that he had periodic stomach aches in childhood. Claimant argued that the assured did not disclose this in the proposal, as he did not consider it a serious enough condition. A doctor applies for life insurance soon after a series of seizures, which he does not disclose in the proposal papers. (A) The first is a case of innocent non-disclosure, while the second is one of deliberate disclosure. (B) The first is a case of innocent misrepresentation and the second one of deliberate misrepresentation. a) Both statements A & B are correct b) Statement A is correct c) Statement B is correct d) None of the above Solution: c 361. Consider the following two cases: Famous paintings were destroyed in a fire accident in an art gallery. A cargo ship carrying 1000 ton grain was partially ruined by dampness while in transit. In both cases the items were insured as valued policies (1) The claims under both instances would be settled for the full value at risk.

(2) The claims in the first case will be settled for the full value at risk while the second case will be settled on indemnity basis. a) Statement 1 is correct b) Statement 2 is correct c) Both cases will be settled on indemnity value basis d) None of the above Solution: b 362. Mr. Sharma is 30 years old and plans to retire at the age of 60. He is employed as the Vice President with Global Cosmetic Ltd. at a monthly remuneration of Rs.55,000. His life insurance premium is Rs.18000 with total sum assured as Rs.12, 00,000. He pays an insurance premium of Rs.10,500 and Rs.6500 for his wife and child respectively. He also pays a professional tax of Rs.3000 and income tax subject to allowable deductions of Rs.1,32,000. The rate of interest assumed for capitalization of future income is as 8%. Calculate Mr. Sharma Human Life Value (HLV) to recommend adequate insurance cover. a) Rs.42 lakhs b) Rs.50 lakhs c) Rs.55 lakhs d) Rs 1 crore Solution: b find pv given pmt=5,07,000; nper = 30 ; rate = 8 % total salary 6.6 lakhs 1.53 [ 1.32 + .03 + .18]= 5.07 lakhs pv = 61.64 lakhs 12 lakhs = 49.64 round off 50 lakhs. 363. Calculate the yearly premium for the data given below: Date of Birth: 19/09/1979 Date of Commencement of Cover: 21/02/2005 Plan Term: Endowment with profits 25 years Sum Assured: Rs.1,00,000 Mode Rebate: Yearly-3%, Half-yearly-1.5% Sum Assured Rebate: Upto Rs.49,999 - Re. 1 per thousand Rs. 50,000 and above - Rs. 2 per thousand Tabular premium: Age 24 - Rs. 49.50 Age 25 - Rs. 51.75 Age 26 - Rs. 52.50 a) Rs. 4,920 b) Rs. 4,998 c) Rs. 5,020 d) Rs. 5,010 Solution: 4820 .51.75 1.5525 =50.1975 -2 = 48.1975 per thousand for 1 lack = 48.20 * 100 = 4820 364. What would be the paid-up value and surrender value as on 20.03.2006 for the data given below? Date of Commencement of Cover: 20.03.1995 Plan Term: Endowment 20 years with profit Sum Assured: Rs. 20,000 Mode: Half-yearly Bonus Accrued: Rs. 800 per thousand SA for 10 years duration SVF: 51.2% a) Rs. 26,000 (Paid-up value) and Rs. 13,312 (Surrender Value) b) Rs. 15,500 (Paid-up value) and Rs. 13,050 (Surrender Value) c) Rs. 25,000 (Paid-up value) and Rs. 12,800 (Surrender Value) d) Rs. 26,000 (Paid-up value) and Rs. 13,568 (Surrender Value) Solution: 20000 * 23/40 = 11500 + 800 * 20 [bonus]= 27500 assuming hes paid march 06 premium unless stated otherwise. SV = 27500 * 51.2 /100 = 14080. 365. From the following data, calculate the amount of claim payable, if the insured dies on 27.10.2005. Date of Commencement: 13/09/1989 Sum Assured: Rs.1,00,000 Plan Term: Money back (with profit) 25 years Last premium paid due: 13/09/2004 Three installments of Rs.20,000 each have been paid in 1994, 1999 and 2004 Assume that bonus declared is Rs.600 per thousand SA, Assume premium: Rs.2,000 d) Rs. 1,00,000 d) Rs. 64,000

d) Rs. 1,24,000 d) Rs. 1,56,000 366. Calculate the premium from the following data: Plan: Money Back 20 years Sum Assured: Rs.1,00,000; rebate for large sum assured Rs.2 per thousand Mode: Half-yearly; rebate- 1.5% Age: 36 years Proposal accepted at ordinary rates with double accident benefit Tabular premium: Rs. 69.25, Premium for double accident benefit is Re. 1 per thousandper annum a) Rs. 3,361 b) Rs. 3,461 c) Rs. 3,463 d) Rs. 3,261 Solution: a 69.25 1.5 % - 2.00 + 1 = 67.21 * 100 /2 367. Calculate half-yearly premium as on 15.10.2006: SA: Rs. 3,00,000 Date Of Birth: 16.06.1982 Date of Maturity: 15.09.2036 Term: 25 years Double Accident Benefit: Re. 1 per thousand Mode Half-yearly Tabular premium o Age 24: Rs. 49.50 o Age 25: Rs. 50.60 o Age 26: Rs. 52.00 d) Rs. 6,575 d) Rs. 7,775 d) Rs. 7,575 d) Rs. 6,775 Solution: c 49.50 + 1 =5050 / 2 * 300 = 7575 368. Calculate surrender value: SA: Rs. 1,00,000 Date Of Birth: 28.03.1985 Last Premium Paid: 28.03.1999 Term: 30 years Bonus: Rs. 800 per thousand SA Surrender Value Factor: o 72% (for 14 years) o 78% (for 15 years) o 80% (for 16 years) Mode of Payment: Quarterly Date of Calculation: 28.06.1999 a) 83,916 b) 82,620 c) 82,600 d) 91,800 369. From the following data, calculate the loan amount that can be given: Date of Commencement: 20.08.1990 Plan Term: Endowment with profit, 25 years Sum Assured: Rs. 1,50,000 Last premium paid 20.08.2003 Surrender Value Factor 55% Bonus Accrued Rs. 1,000/1,000 a) Rs.1,16,000 b) Rs.1,19,900 c) Rs.1,20,500 d) Rs.1,15,800 Solution: d 150000 * 14/25 = 84000 + 150000 [ bonus]=234000 * 55/100 128700 * 90 / 100 [ loan at 90 % of SV ]= 1,15,830. 370. From the following data, calculate the yearly installment: Sum Assured: Rs.1,00,000 Plan Term: Endowment Plan 20 years

Date of Maturity: 23.12.2022 Date of Birth: 28.06.1974 Tabular premium o Age 27: Rs. 26.93 per thousand o Age 28: Rs. 27.83 per thousand o Age 29: Rs. 28.37 per thousand Premium Adjustment: o 5% extra for monthly mode o Re. 1.00 less for half-yearly mode o Rs. 1.50 less for yearly mode Rebate of Rs. 1.50 for sum assured of Rs. 50,000 and above Assume premium will be paid annually a) Rs. 1,241 b) Rs. 1,242 c) Rs. 2,483 d) Rs. 2,843 Solution: c 27.83 3 [ 1.5 disc for yearly mode + 1.5 SA discount]= 24.83 371. Financial Quantifiable Loss is termed as _____________. a) Speculative Risk b) Financial Risk c) Personal Risk d) Monetary Risk Solution: b 372. The direct advantage of insurance is a) Indemnification for unexpected losses b) Indemnification for expected profits c) Safety of funds d) Elimination of occurrence of insured risk Solution: a 373. Mr. Shah took a Life Insurance Policy, which he continued for 3 years. He died within a night thereafter, suffering from cancer. Mrs. Shah lodged the claim as Nominee. The sum assured was Rs.10 lakh. The insurer had indisputable evidence that Mr. Shah was hospitalized for 20 days earlier to his taking the proposal which was not disclosed in the proposal form. Though no fraudulent was established, the insurer repudiated the claim because _________________. a) Repudiation is in keeping with Law. b) Repudiation is not in keeping with the Law. Solution: b 374. Malhotra Heart Centre has a great marketing centre and a full business. Dr. Kohli, the Chief Heart Surgeon is held in high esteem nationwide for his expertise and success rate in surgery. He has made an outstanding contribution for the last 8 years to enable market leadership for Malhotra Heart Centre. The most appropriate policy that the owners may take in the interests of business on the life of Dr. Kohli would be _____________. a) Personal Accidental Insurance b) Term Insurance c) Key Man Insurance d) Mediclaim Insurance Solution: c 375. Shah & Sons had a Fidelity Guarantee cover for its cashiers. The cashier committed a fraud in April 2002. The policy expired in the month of March 2003. Neither was it renewed. Fraud was detected in December 2003. Claim was lodged. The claim is _____________. a) Payable b) Not Payable c) Payable partially Solution: b 376. A Prospect and Planner agreed for an Endowment Assurance Policy for Rs. 4 lakhs, limited payment for 10 years with the term of 20 years. If Reversionary bonus is taken as 7.5% p.a and terminal bonus as Rs. 150/-per 1000/-. The maturity value would be ______. g) Rs. 10, 50, 000 g) Rs. 10, 60, 000 g) Rs. 10, 70, 000 g) Rs. 10, 80, 000 Solution:b Revisionary Bonus = 30,000 * 20 = 6.00 lakhs Terminal Bonus 0.60 lakhs SA 4.00 lakhs ---------------Maturity value 10.60 lakhs

377. Mr. Khanna will retire 6 months from now completing 65 years. His 2 daughters are residing abroad. He and his wife both have good health and live in a flat. Liabilities are insignificant and post retirement benefits substantial. The flat is very well decorated with valuable movable and immovable items. He purchased a car last year. He does not have any insurance cover except for the car, as of now. The most relevant cover for him would be __________. d) Endowment Assurance d) Money Back d) Household Insurance d) Term Insurance Solution: b 378. The correct priority sequence in steps of review are_________. 1. Establish a frequency. 2. Establish change in clients circumstances. 3. Develop revised program. 4. Take approval of the client to revised program a) 1234 b) 1342 c) 1324 d) 1423 Solution: a 379. The advantage of insurance mechanism include a) Indemnification for unexpected losses b) Indemnification for expected profits c) Safety of funds d) Elimination of chance occurrence of insured risk Solution: a 380. Himanshu owns a flat worth Rs.2,00,000 (market value). He has insured it only for Rs.1,50,000. The flat is damaged by an earthquake, and loss is assessed at Rs.1,00,000. Assuming the insurer applies the principle of Average to the loss the insurer will get a claim payment of _________. a) Rs. 75,000 b) Rs. 70, 000 c) Rs. 37, 900 d) Rs. 1,00,000 Solution: a because under insured by 25 % 381. The principles of Risk management are applicable to a) Individuals b) Small Enterprises c) Huge Commercial enterprises d) All the above Solution: d 382. The two elements involved in evaluation of risks includes probability a) of loss producing events occurring and potential losses b) of occurrence of events. c) of rising inflation on future income and cost of assets. d) of increasing cost of operation and employment Solution: a 383. What is risk control? a) Anticipation, ascertaining and increasing the chances of occurrence of loss producing events b) Avoiding, eliminating and reducing the chances of occurrence of all events. c) Avoiding, eliminating and reducing the chances of occurrence of loss producing events or d) Anticipating, asserting and increasing the chances of occurrence of loss. Solution: c 384. Who normally makes the offer in case of Insurance? d) Agent d) Insurer d) Prospective buyer d) Anyone Solution: c 385. The insurance contract promises to a) Compensate in case of insured contingency b) Pay in case of any event c) Pay on demand d) Pay the premiums Solution: a 386. The insurance contract is between a) Insurance company and any person. b) Insurance company and its agent c) Insurance agent and the Insured client.

d) Registered insurer and the insured (individual or firm) who is competent to contract. Solution: d 387. What time does Insurable interest exist, in the case of property Insurance? a) At the time of entering contract b) At the time of loss occurring c) At any time d) Both at the time of entering into the contract and at the time of loss Solution: d 388. Insurable interest is not satisfied between whom under the Life Insurance contract? a) Husband-Wife b) Parent-Child c) Friends d) Employer and Employee Solution: c 389. What does a Pure Risk involve? a) Loss or No Loss b) Loss and Gain c) Gain d) All the above Solution: a 390. How much amount can the insurer recover under the existing subrogation rights? a) Equivalent to the claim. b) Equivalent to the sum insured c) Not more than 1 lakh d) 20 % of sum insured Solution: a 391. Material fact has been defined as the fact that would a) Influence the mind of insured b) Influence the mind of underwriter in declining or accepting the risk c) Influence the mind of other policyholders d) Not affect the insurance contract Solution: b 392. An example of material fact under Life insurance is a) Occupation, health and habits of the person to be insured b) Particulars possessions of the person c) Particulars about the vehicle owned d) None of the above Solution: a 393. Insurance a) Protects the loss of an asset due to peril b) Compensates the loss caused by a peril c) Safeguards the loss of an asset due to peril d) None of the above Solution: b 394. Violation of Utmost Good faith gives the aggrieved party d) The right to avoid the contract d) The right to nondisclosure of breach d) The right to honour the contract d) None of the above Solution: a 395. The term used to alter the insurance contract is _________. a) Deductibles b) Policies c) Endorsements d) All of the above. Solution: c 396. In co-insurance, the claims of the policy is shared by a) All insurers in the same proportion as their share bears to the total value insured b) Insured and insurers c) Total value insured will be shared by main insurer d) Main insurer and the insured Solution: a 397. Insurance means a) Sharing of risks b) Sharing of liability c) Sharing of responsibility d) None of the above Solution: a 398. Life insurance is primarily a) An investment tool b) A financial security tool c) A tax saving tool d) None of the above Solution: b

399. _____________ are insurable. a) Speculative Risk b) Financial Risk c) Gambling Risk d) Pure Risk Solution: d 400. The insurance mechanism is used to a) Control the risk b) Transfer the risk c) Retain the risk d) All of the above Solution: b 401. Insurance covers a) Dynamic Risk b) Speculative Risk c) Pure Risk d) Fundamental Risk Solution: c 402. Peril refers to a) Wear and tear of assets b) Natural calamities c) Accidental occurrences d) Either B or C Solution: 403. The mechanism of insurance is a) Reimbursement of losses by the Government b) Channeling of savings into investments c) Charring of risk by the community to minimize impact on a single member d) None of the above Solution: 404. Material facts should be disclosed during a) Proposal stage b) Revival stage c) Alteration d) Any of the above Solution: d 405. Uberrima fides means a) Utmost Good Faith b) Ad idem c) Caveat Emptor d) Doctrine of warranty Solution: 406. Without insurable interest, an insurance contract would be a) Void-ab-initio b) Wagering contract c) Void contract d) Commercial contract Solution:b wagering is basically betting / gambling . 407. Which of these is not an example of automatic insurable interest a) Husband and wife b) Brother and sister c) Company and a key-executive d) Any of the above Solution: b 408. Which material facts need not be disclosed? a) Facts of law b) Facts of common knowledge c) Facts known only to the proposer d) Both A and B Solution: 409. In order to be insurable, the risk should be a) Capable of financial measurement b) Similar to a large number of homogenous risks c) Not against public policy d) All of the above Solution: d 410. The components of Personal Risk that are faced by the individual is/are a) Death b) Disability c) Medical d) All the above Solution: 411. A material fact is a) A fact that would influence the judgment of a prudent insurer in fixing the premium or determining

b) A matter of common knowledge c) A fact that will reduce the premium d) A fact that will increase the premium Solution: a 412. In case of whole life insurance, the sum assured is normally paid to the a) Life assurer b) Nominee c) Legal heir d) Any of the above Solution: b 413. The premium charged by the life insurer is calculated on the basis of a) Mortality Rate b) Interest c) Expenses d) All of the above Solution: d 414. Which of the following are types of annuity? a) Immediate b) Deferred c) Flexible d) All of the above Solution: d 415. What is Reinsurance? a) Distribution of insurance business made by the insured. b) An agreement concluded by one insurer with another. c) The other name given to coinsurance d) The action of insured in getting the same risk insured by another insurer. Solution: b 416. Name the area(s) under which legal liability risks may arise. a) Under Status b) Under Contract c) At Common Law d) All of the above. Solution: d 417. The damages that are awarded during personal injury claims include a) Special damages and General damages b) Special damages only c) Damages to third parties only d) None of the above Solution: a 418. What is the liability that arise due to failure of one of the parties not fulfilling the provisions of contract which it had entered into with another party? a) Vicarious Liability b) Special Damages c) Contractual Liability d) Strict Liability Solution: c 419. Life Insurance is a) Total loss cover b) Indemnity cover c) Substitute for loss of income d) All of the above Solution: c 420. Economic value of an individual includes a) Self b) Self and dependents c) Family (Economic unit) d) Only dependents Solution: c 421. Insurable Risks are categorized into a) Private and Official Risk b) Personal and Property and Liability Risks c) Official and Property Risks d) Private, Official and Property Risks Solution: b 422. Human Life Value in general is a) Total income of an individual b) Total wealth of an individual c) Earning capacity d) None of the above 423. Human Life Value amounts to a) Total savings b) Total investments

c) Assets + Estimated future economic value of an individual his family d) Past income 424. Why does an office conduct a special enquiry in the case of death claim which arose after 4 years? a) As a precaution b) for a large sum assured c) As a routine d) On suspicion of suppression of facts Solution: a 425. Which of the following statements is/are True? Statement A: The fact of having been on a sick leave must be disclosed even if there were no sickness and the certificate of sickness was false. Statement B: Most plans of insurance are a combination of two basic plans. a) Only statement A b) Only statement B c) Both statements d) neither of the statements Solution: a 426. Bringing a policy, which has lapsed, back to force is called___________ a) Recovery b) Renewal c) Revival d) Relapse Solution: c 427. Which of the following statement(s) is are True? Statement A: Life insurance policies are available through the Unit Trust of India also. Statement B: In a joint life policy, death claim is payable only when both the insured die. a) Only statement A b) Only statement B c) Both statements d) Neither of the statements 428. A persons insurable interest in his own life is ___________ a) Unlimited b) 10 times his salary c) Equal to his assets d) Obligation for family Solution: a 429. Which of the following statement(s) is/are True? Statement A: Sick people may be considered for grant of insurance on special terms. Statement B: Insurance cover begins as soon as the premium is received in the office along with the proposal. a) Only statement A b) Only statement B c) Both statements d) Neither of the statements Solution: c 430. The behaviour of an agent who tells his client that the advice given by another agent is wrong is _______ a) Non-professional b) Interfering c) Childish d) Dominating Solution: a 431. What risk is covered by an annuity? a) Sickness b) Accident c) Living too long d) Death Solution: c 432. What measures are taken to reduce or eliminate the chances of occurrence of loss? a) Control Measures b) Cost Measures c) Administrative Measures d) All the above Solution: a 433. The impact of the death of one sole breadwinner of the family over 40s family would be severe in the area of a) Meeting outstanding debts b) Meeting basic needs c) Meeting medical needs d) Meeting educational needs Solution: b 434. XYZ Ltd has a project that is insured for 10 crores. 60% of its risk is covered by Insurance company A while 40% of the balance risk is covered by Insurance company B. The insurance policy is issued by Insurance

company A. It has a collective insurance clause setting out the liability of both the insurance companies. This case is one of a) Co-insurance b) Reinsurance c) Both (a) and (b) d) None of the above Solution: a 435.Rahul and Priya (husband and wife) were born in 1950 and 1958 respectively. Rahuls life expectancy at birth is 65 years while Priyas life expectancy at birth is 72 years. Rahul is planning to buy an annuity that will be paid to him or his wife till any one of them is alive. For how long should he purchase this annuity for? a) 10 years b) 12 years c) 17 years d) 22 years Solution: d 436. Venkat proposes an insurance cover for his farmhouse which is occupied at intervals during the year. Which of these may be excluded from coverage by the insurance company? a) Fire b) Theft c) Third party liability d) Loss of rental income Solution: b 437. Naresh is a car driver by profession and drives Harishs car. Naresh is involved in a collision with the boundary wall of a neighbours house and injures the neighbour as well. Harish was also in the car during the collision. It was later discovered that Naresh was under the influence of alcohol. While setting the claim, which of these perils will not be covered? a) Repair charges to Harishs car b) Boundary wall repair charges to the neighbours house c) Medical treatment charges of Harish d) Medical treatment charges of Rahul Solution: c 438. Vikram, an insurance advisor advises his client, Mr. Singh about the purchase of insurance plans for tax exemptions which will earn Mr. Singh a 50% rebate. However, Mr. Singh was unable to obtain this tax redemption and plans to sue Vikram for giving him misleading advice. Which of these methods can Vikram adopt as recourse? a) Asset Insurance b) Legal Liability Insurance c) Comprehensive Insurance d) Professional negligence cover Solution: d 439. Victor is 24 years old and a young executive who has begun his career. He plans to start a family in 3-4 years. He travels extensively by road and out of the city. What kind of life insurance cover would you recommend for him? a) Endowment insurance policy b) Money back policy c) Unit linked policy d) Temporary insurance policy with convertibility and renewability option Solution: d 440. Mr. Sharma takes out a policy for his son Rahuls higher educational needs after Rahul reaches 18 years of age. Unfortunately, after paying a premium for 5 years, Mr. Sharma passes away. How can the policy be enforced without paying future premiums so that the intended benefits are preserved? a) Accident benefit premium b) Line on policy c) Premium waiver facility d) None of the above Solution: c 441. Mr. Virendra Tripathi is an executive in an Indian company. He is in his early 30s and has a baby girl. He plans to sign up for a flat immediately. His spouse is not working. Mr. Tripathi has no life insurance yet. Which of these combinations would you recommend for his case? a) Limited payment endowment assurance, Joint life, Childrens deferred, Health b) Term insurance, Joint life, Childrens deferred, Mortgage redemption c) Whole life with term assurance for spouse, Educational annuity with premium waive d) Money back policy, Childrens deferred policy, Joint life policy, Mortgage redemption Solution: c 442. The principle of insurance is based on a) Sharing of Resources b) Sharing of Losses c) Sharing of Capital d) Sharing of Investments Solution: b

443. Please see the following data about life expectancy at birth of men and women during the last three decades in India-Census Year Male Female 1971 46.40 44.70 1981 54.10 54.70 1991 58.10 58.60 2001 62.30 66.27 Certain conclusions are drawn below based on this study. Which of these conclusions CANNOT be deduced from this data? a) Today, female children enjoy longer lives than those born three decades ago b) Womens mortality has improved more than the mortality experience of men during the last c) The same trend as above is seen in the last two decades as well d) The mortality experience of men has improved in comparison to women in the last decade Solution: d 444. Consider the following situations1. Mr. Sinha, aged 30, is a Major in the Indian Armed forces. His proposal and family history reveal that he suffered from typhoid at the age of 18 years. His father passed away a year ago at the age of 70 due to a heart attack. The underwriter will not consider any of these features as adverse. 2. Mr. Bajaj, aged 35, is overweight by 10% and consumes one litre of beer every alternate day. His parents are alive. The underwriter is likely to ignore these features. a) Only Statement A is correct b) Only Statement B is correct c) Both statement A and B are correct d) Both statement A and B are incorrect Solution: d 445. Read the following two statements: (I) Death of the breadwinner is a Financial Risk (II) It is a Pure Risk a) I & II are correct b) I & II are incorrect c) I is correct d) II is correct Solution: a 446. Ram lost his lifes savings when the owners of the private financial company, where he deposited it, went bankrupt due to questionable practices and defaulted on repayment. It is the case of loss under ___________. a) Investment Risk b) Speculative Risk c) Regulatory Risk d) None of the above Solution: a 447. Hillary had a choice of investing her retirement dues in post office savings or bank deposit or with private fund raiser. She barely manages to get her funds back when she opted for the third option. This is the case of ___________. a) Not understandings the risks b) Taking a gamble for high returns c) Succumbing to greed d) Exercising poor judgment Solution: a 448. Mr. Jain has suffered several damages to his property (uninsured) because of fire. Thereby he suffered several financial losses. This happened because he did not cover _________. a) The Investment plan b) The Pure Risk c) The Speculative Risk d) None of the above Solution: b 449. Tarun, a driver, caused injuries to a pedestrian by rash driving of car. The injured victim had to spend Rs.1000 in treating his injuries. Taruns act has created liabilities under _________. a) Common Law b) Contract c) Statute and Common Law d) Statue only Solution: c 450. Mr. Roy took a business premises on lease with the provision that he himself had to pay the insurance premium for fire and other perils and not the owner of the premises. This would be an instance of __________. a) Risk Control b) Risk Transfer c) Risk Retention d) Risk Reduction Solution: b

451.Case 1: An insurer receives an application for insurance which contains an incomplete or missing answer. The insurer does not contact the applicant for the missing information and the policy is issued, subsequently when claim arose under the policy, payment was denied on the basis of the earlier incomplete information. Case 2: In another case an insureds car insurance renewable premium was about to fall due. On enquiry with the company, they told him that there was 15 days grace period from that day during which time period he could pay. Subsequently, the car met with an accident next day, the company denied the claim on the ground that the premium was not paid on time. (I) The principle of estoppel and that of waiver in the first and second case respectively makes the denial of claim untenable. (II) In the former, the principle of waiver and in the latter, the principle of estoppel were violated, because of which the denial of claims were set aside by courts. a) A is correct b) B is correct c) A & B are cases of aleatory contact d) None of the above Solution: b 452. An insurable loss must not be a) Catastrophic b) Fortuitous or accidental c) Definite d) None of the above Solution: a 453. After two years of taking out of life insurance policy, the assured dies after complications from emergency surgery after an acute attack of duodenal ulcer. Enquiries revealed that he had periodically stomach aches in childhood. Claimant argued that the assured did not disclose this in the proposal, as he did not consider it as a serious thought enough condition a doctor applies for life assurance soon after a series of seizures, which he does not disclose in the proposal papers. (A) The first case of innocent non-disclosure, while the second is of deliberate non- disclosure. (B) The first is a case of innocent misrepresentation and second one of deliberate misrepresentation. a) A & B are correct b) A is correct c) B is correct d) None of the above Solution: b 454. A fire insurance covered premises was housing a constructors office. In between, the constructor vacated the premises and the space was rented out as a clothing store. The insurance company was not informed about the change of the tenant. The grounds on which the insurer refuses to entertain the claim, if the premises suffer a fire would be __________. a) Doctrine of utmost good faith. b) Doctrine of adheres ion. c) Principle of duty of disclosure d) Principle of indemnity Solution: c 455. Showroom of Rajesh & Sons, Drycleaners caught fire in an accident where many items of clothing of customers were destroyed. The contents were insured. The owners claim for damage of clothes etc. will be considered by the insurance company under _________. a) Property Liability b) Financial Liability c) Legal Liability d) None of the above Solution: c 456. Which of the following contracts do not adhere to the application of law contract? ) Insurance ) Stock Market ) Property Deals ) None of the Above Solution: d 457. Insurance contracts can be __________ contracts. d) Indemnity d) Negotiated d) None of the above d) Either A and B Solution: d 458. Peter has insured his two-wheeler for a property risk. He wears a helmet while he drives but does not have an Accident Insurance. He has ______________. d) Taken a grave risk by not taking accidental insurance. d) Made a mistake, he should have taken Accidental Insurance instead of property risk insurance. d) Transferred his property risk, controlled some of the risks and retained the rest. Solution: c d) Done a good thing by reducing his premium payout personal by not taking Accidental Insurance.

459. Mark is an advocate having a roaring practice consisting of highly valued intricate cases of companies. He has 3 young juniors. Cash transactions do not take place in office. He maintains his office with modest furnishings. The most necessary cover for him is d) Fidelity Guarantee d) Professional Indemnity d) Public liability d) Burglary Solution: b 460. An insurance party reinsures the risk with a re-insurer, this happens in the case of d) Risk retention d) Risk Transfer d) Both A and B d) None of the above Solution: b 461. Mr. Manish insures his home worth Rs.50 lakhs for Rs. 30 lakhs. The house was destroyed in fire and he suffers loss worth Rs. 20 lakhs. How much will he receive from the Insurance Company? d) 20 lakhs d) 16 lakhs d) 12 lakhs d) 30 lakhs Solution: c 462. An insurable loss must be d) Certain d) Fortuitous or accidental d) Indefinite d) Impossible to happen Solution:b 463. Mr. Mohammed, 30 years and married, works for a firm which provides him with medical cover. He already has his own home and savings of Rs.42 lakhs which are well invested. In the next twenty years, he will be able to save enough to fund his retirement and his childrens education. Which of the following might be the most important insurance for him? d) Medical Cover d) Temporary Total Disablement Cover d) Property insurance d) Life Cover Solution: d 464. Sam got some injuries at the building site when the building contractors car ran over his toe. He was unable to attend work for 6 weeks. What are the options available to Sam? d) Sam and the contractor get the Accident Insurance. d) Sam gets Accident Insurance. The contractor gets third party Liability Insurance d) Sam gets Disability Insurance, if he has one. The Contractor gets a third party Liability Insurance d) Sam gets Disability Insurance, and the contractor gets Accident Insurance. Solution: b 465. Mark, a Manager earning 40,50,000 p.a has the following details: Electronic items market value 3, 75,000 Acquisition value 37,500 Furniture replacement cost 21,00,000 House market value 97,50,000 Cost of building house 22,50,000 + cost of the land Daughters marriage 3,75,000 Childrens education 2,25,000 Retirement funding 6,75,000 Current professional Indemnity. 2,25,000 Current investments 15,00,000 What should be his priority in regards of insurance to the following? 1. PTD 2. Life cover 3. Property insurance 4. Professional Indemnity j) 1,2,3,4 j) 2,3,1,4 j) 3,1,2,4 j) 4,3,1,2 Solution: a 466. Mr. & Mrs. Sharma, aged 50 and 47, both have a life expectancy of 35 years. Calculate the insurance required based on need based and income replacement methods on Mr. Sharmas life. You have the following information: Current investments Rs. 25,00,000 Expenses Rs. 3,00,000 (including 1 lakh of Mr. Sharmas personal expense) Mr. Sharmas income post tax Rs. 3.5 lakhs Final costs Rs.1 lakh

Post tax, post inflation rate/discount factor is 3% d) 42 lakhs, 117 lakhs d) 20 lakhs, 68 lakhs d) 42 lakhs, 83 lakhs d) 20 lakhs, 52 lakhs Solution: Need based: Pmt = 200000; nper = 35 ; rate = 3 % pv= 44.26 lakhs add 1 lakhs final cost and deduct 25 lakhs investment 44.26 + 1 25 = 20.26 lakhs income replacement[retention] = 2,50,000 * 100/ 3 = 83.333 lakhs 467. Mr. & Mrs. Arora have Mr. Aroras father staying with them, who is entirely dependent on them. As an insurance advisor, you would estimate the life expectancy of _________. e) Mr. Arora because he manages the expenses. e) Mrs. Arora because if she is not there, who would look after the father. e) Mr. Aroras father because Mr. & Mrs. Arora have to support him for life. e) None of the above Solution: c 468. Puneet, aged 50, works for a private firm. Most of his earnings is through commission. He does not have benefits like sick leave, LTA etc., he is single and already had TPD cover. His priority should be ____________. d) Life cover d) Health insurance d) Endowment policy d) Property insurance Solution: b 469. Insured Declared Value (IDV) does not apply to _______________. d) Property damage in motor accident d) Curios, old jewellery d) Third party liability claim d) Property insurance Solution: c 470. Mr. Dutta lives in Kerela, which is not earthquake prone. He decides to take out insurance on his property. As an insurance advisor, you would advise him that _________. d) He should not take out property insurance since he does not need it. d) He should not take the earthquake option, since it will be expensive d) He should not take out property insurance, since it will be expensive d) He can take out property insurance as the premium will depend on the risk and will therefore Solution: c 471. An insurance company makes a surplus, due to _______________________. 1 Mortality of policy 2. Investment rate 3. Expenses 4. Lapse, surrenders 5. Adverse loss ratio d) 1,2, 3 d) 1,2,3,4 d) 1,2,3,4,5 d) None of the above Solution: a 472. Because of the application of the doctrine of adhesion, the courts construe the terms of insurance contract in favor of d) The insured d) The insurer d) None of the above d) Depends on circumstances Solution: d 473. Consider the following statements: (A) Payment of the first premium is legal consideration (B) Payment of the first years premium as also the subsequent two years premium constitute legal consideration. d) Only Statement A is correct d) Only Statement B is correct d) None of the above. d) Statements A and B are both incorrect Solution: d 474. Consider the following statements: (A) The proposer has to demonstrate utmost good faith while proposing for insurance (B) Mere good faith is enough. d) Statements A & B both are correct d) Statement A & B both are incorrect

d) Statement A is incorrect, while B is correct d) Statement A is correct & B is incorrect Solution: c 475. Risks must be __________ before they can be _________ d) severe, assessed d) identified , measured d) measured, identified d) none of the above Solution: d 476. There is an Excess Clause of Rs 5000 in a health insurance policy. So when a genuine claim is filed for Rs 50000, In such case the insurance company will pay _________. d) Rs 5000 d) Rs 45000 d) Rs 50000 d) Rs 55000 d) None of the above Solution:b 477. A person has expenses relating to a loan of Rs 2 lakh, medical expenses of Rs 50000, education expenses of Rs 2 lakh and other domestic expenses relating to 1 lakh. With an annual income of Rs 7 lakh and a 5 time income multiple the insurance need according to the income method is a) Rs 450000 b) Rs 500000 c) Rs 700000 d) Rs 3500000 Solution: d 478. In the above case the requirement as per the income plus expense method is a) Rs 3000000 b) Rs 3500000 c) Rs 4050000 d) Rs 4500000 Solution: c 479. Mr. Rajan has an outstanding debt of Rs 2 lakh, medical expenses of Rs 0.50 lakhs per annum, education expenses of Rs 2 lakh per annum and other self maintenance expenses of Rs.1 lakh per annum plus he earns an annual income of Rs 7 lakh, As a Financial Planner find out his Insurance Requirement as per Income Rule (Note :- A Five time income multiplier is the accepted industry norm) e) Rs 7,50,000 e) Rs 12,50,000 e) Rs 10,00,000 e) Rs 35,00,000 e) None of the above Solution: d 480. Refer to the case above, as a Financial Planner find out his Insurance Requirement as per Income Plus Expense Approach e) Rs 35,00,000 e) Rs 39,50,000 e) Rs 40,50,000 e) Rs 41,00,000 e) None of the above Solution: c 481. To whom the principles of risk management are applicable? a) Individuals b) Small enterprises c) huge commercial enterprises d) all the above Solution: d 482. Which are the two elements involved in evaluation of risks? a) Probabilities of loss producing events occurring and potential losses b) Probabilities of occurrence of events c) Probability of rising inflation on future income and cost of assets d) Probability of increasing cost of operation and employment Solution: a 483. Which of the following is not an example of Risk reduction when driving? d) Wearing a seat belt d) Driving within speed limits d) Driving defensively d) Not driving Solution: d 484. Which of the following is/are considered to be pure risk? d) Property risk d) Liability risk d) Risk from failure of others d) All the above Solution: d

485. Rama owns a property worth Rs. 45 Lakh. She could only afford to pay a premium, which can insure her property to the extent of Rs. 35 Lakh. This is an example of : e) Normal Insurance e) Over Insurance e) Double Insurance e) Under Insurance Solution: d 486. The formula used to calculate the sum payable to the insured where a subject to average clause is mentioned in the contract is: d) Loss*(Sum Insured/ actual value) = sum payable d) Actual Value*(Sum Insured/ loss) = sum payable d) Sum payable/(loss*sum Insured) = Actual Value d) All the above Solution: a 487. Ram got an Insurance from a company, which stood in his place in relation to a claim on a 3rd party. This example best relates to: d) utmost good faith d) un-insurable interest d) subrogation d) non-indemnity Solution: c 488. Raj has insured his house with a well known insurer. He did not inform the insurance company that he has stored fire-works in his house. In this case he has failed to disclose d) relevant fact d) material fact d) known fact d) common fact Solution: b 489. When evaluating a loss exposure, all of the following projections are made EXCEPT: ) Monetary value of a specific loss ) Total dollar loss of all losses during a particular time period. ) Frequency of losses during a particular time period. ) Location of the loss. Solution: d 490. Which of the following statements concerning range is/are correct? I. Range equals the highest value minus the lowest value. II. Uses all values in the data set. d) I only d) II only d) Both I and II d) Neither I nor II Solution: a 491. Jack was very bored on a rainy day. He spent most of the afternoon randomly drawing cards to see how often an ace appeared. He drew a total of 1,050 cards. During that time, an ace appeared 84 times. What is the relative frequency of Jack's drawing an ace? d) 1.25% d) 8% d) 12.5% d) 15% Solution: b 492. Which of the following is an advantage of pre-funding a loss? d) Money is free for other uses until needed for losses d) Costs can be spread over a period of time and limited to the exact amount required to pay for the losses d) Money in reserves cannot be used to support other activities d) Reserves can be built up over many budget periods Solution: b 493. The human life value approach is based on a) a persons eligibility for social security income b) the amount of life insurance he or she owns c) the standard of living he or she wishes to have d) the capitalized present value of potential lifetime earnings Solution: d 494. Which of the following is a characteristic of an ordinary life policy? a) It is the most expensive form of cash value insurance. b) It matures at age 65. c) It has an increasing cash value and decreasing risk amount. d) Both the cash value and amount at risk increase annually. Solution: c 495. Which of the following statements concerning yearly renewable term is correct? a) The insurance company may experience some adverse selection at renewal time. b) The premiums do not increase from year to year. c) Evidence of insurability must be furnished at the time of each renewal.

d) The insured can renew the policy each year by completing a medical exam. Solution: b 496. In case of insurance who should normally make the offer? a) agent b) insurer c) prospect d) anyone Solution: c 497. What is the binding force of any contract? a) offer b) acceptance c) parties d) consideration Solution: d 498. What is consideration that insurer makes under insurance contract? a) promise to compensate in case of insured contingency b) promise to pay in case of any event c) promise to pay on demand d) promise to pay the premiums Solution: a 499. Which insurance contracts are not contracts of indemnity? a) Fire b) Marine c) Life Insurance d) Motor Solution: c 500. A warranty in insurance contracts are a) Guarantee b) Stipulations imposed by insurer because he wants to ensure that the risk remains same throughout the contract and does not increase c) Guarantees to increase the risk in case of default on part of insurer d) Guarantee to pay the loss to the insurer in case of the loss being higher than the sum insured. Solution: b 501. What are used when the terms of insurance contracts are to be altered? a) deductibles b) endorsements c) policies d) all the above Solution: b 502. __________ are the provisions under insurance contracts that stipulate deduction of a pre-specified amount from the claims payable under the policy a) endorsement b) rules c) deductibles d) none of the above Solution: c 503. ___________ is a compulsory deductible from each and every claim that the insurer pays to the insured a) premiums b) excess c) premiums and excess d) franchise Solution: b 504. A condition that increases the chance of loss is called a (n) ) Direct loss. ) Peril. ) Indirect (consequential) loss. ) Hazard. Solution: d 505. A publishing company solicits manuscripts for publication. The publishing company is concerned that an author might plagiarize material and that the person who was plagiarized might sue the publisher. To address this risk, the contract with the author includes a hold-harmless agreement. Through this agreement, the author, rather than the publisher, is held liable for plagiarism. In this situation, the publisher is using the holdharmless agreement as what type of risk treatment measure? d) Risk selection d) Risk avoidance d) Risk transfer d) Risk retention Solution: c 506. A risk that affects the entire economy, or a large number of persons or groups within the economy, is called a (n) d) Objective risk. d) Particular risk.

d) Speculative risk. d) Fundamental risk. Solution: d 507. All of the following are burdens of risk on society EXCEPT d) Risk forces individuals to practice loss control. d) Risk requires reserve funds to be set-aside in case a loss occurs. d) Risk creates fear and worry. d) Risk deprives society of certain goods and services. Solution: a 508. Deans Discount Store has been experiencing problems with shoplifting losses. Dean decided to install a camera monitoring system and to use magnetic price tags on products. If a tag is not demagnetized before the product bearing the tag leaves the store, an alarm bell sounds. These measures are examples of d) Risk transfer. d) Loss control. d) Risk avoidance. d) Risk retention. Solution: b 509.The spreading of losses incurred by a few individuals over a larger group, so that average loss is substituted for actual loss, is known as d) Fortuitous loss. d) Pooling of losses. d) Transfer of risk. d) Loss indemnification. Solution: b 510. All of the following are ideal requirements that must be met for a risk to be privately insurable EXCEPT d) There must be a large number of similar exposure units. d) The chance of loss must be calculable. d) Losses should not be catastrophic. d) The loss should be within the insureds control. Solution: d 511. Which of the following statements is (are) true with respect to insurance and gambling? . Insurance is simply a wager that if a loss occurs, youll be paid for the loss. . Both gambling and insurance are socially productive. d) Both I and II d) II only d) Neither I nor II d) I only Solution: c 512.Chaya does not own health insurance. For the past two weeks, she has been experiencing sharp abdominal pain. Given her condition, she would like to purchase health insurance. When higher-than-average risks (like Chaya) are insured at average premiums, losses are higher than anticipated. What is this problem called? d) Adverse selection d) Morale hazard d) Speculation d) Moral hazard Solution: a 513. The first step in the risk management process is to d) Implement and administer the program. d) Select the appropriate techniques for handling losses. d) Identify potential losses. d) Evaluate potential losses. Solution: c 514. All of the following risk treatment techniques are classified as risk control methods EXCEPT d) Avoidance. d) Loss prevention. d) Loss indemnify. d) Loss reduction. Solution: c 515. Which of the following statements is (are) true with respect to the objectives of risk management? ii.A pre-loss objective of risk management is the reduction of uncertainty. ii.A post-loss objective of risk management is the stabilization of earnings. d) Neither I nor II S d) Both I and II d) I only d) II only Solution: b 516. Bajaj Company manufactures electronic components. Managers of the company are considering several diversification options. One possibility is production of prescription drugs. When Bajaj Company managers learned of the potential legal liability that could result from the manufacture and sale of prescription drugs, the managers rejected the idea and decided to consider other diversification options. How did Bajaj Company choose to deal with the risk of legal liability arising from the manufacture and sale of prescription drugs?

d) Loss control d) Risk transfer d) Risk retention d) Risk avoidance Solution: d 517. All of the following are methods used to pay retained losses EXCEPT d) Borrowed funds. d) Insurance. d) Current net income. d) Funded reserve. Solution: b 518. Which of the following is most likely to occur in a hard insurance market? d) Low insurance premiums and loose underwriting standards d) High insurance premiums and tight underwriting standards d) Low insurance premiums and tight underwriting standards d) High insurance premiums and loose underwriting standards Solution: b 519. Jonathan believes there is a relationship between the number of miles driven by his companys delivery vehicles and the number of physical damage claims that will occur. Jonathan collected data on the number of claims and the number of miles driven for the past 15 years. Using a computer, Jonathan determined the coordinates of a line that best fit these data. Armed with this information, he can predict how many losses will occur next year. The type of analysis that Jonathan employed is called d) Regression analysis. d) Probability analysis. d) Cash flow analysis. d) Time value of money analysis. Solution: a 520. In analyzing the probability that certain events will occur, a risk manager knows that certain events cannot occur together because the occurrence of the first event precludes the occurrence of the second event. Such events are called d) Mutually exclusive events. d) Unconditional events. d) Dependent events. d) Independent events. Solution: a 521. Olivia is Risk Manager of ABC Company. She is trying to determine if a loss control investment is justified. Olivia calculated the present value of the future cash flows she expects the project will generate. She summed the present value of the future cash flows and then subtracted the cost of the loss control equipment. The resulting value is called the projects ) Capital budget. ) Net present value. ) Internal rate of return. ) Loss distribution. Solution: b 522. All of the following are benefits of insurance to society EXCEPT d) Provides a pool of investment funds. d) Enhances credit. d) Reduces insurance company operating expenses. d) Indemnifies losses. Solution: c 523. Kavita opened a sporting goods store. After a fire damaged the store, Kavita was forced to close the business for four weeks while repairs were completed. The loss of profits that could have been earned if the business had remained open is best described as a (n) d) Hazard. d) Peril. d) Indirect (consequential) loss. d) Direct loss. Solution: c 524. An insurance contract must be accepted in its entirety and any ambiguity in the contract is construed against the insurer. Because of these characteristics, we can describe insurance contracts as d) Aleatory contracts. d) Contracts of adhesion. d) Bilateral contracts. d) Valued contracts. Solution: a 525. In which part of an insurance contract would you find information about the property or activity to be insured? Conditions Insuring agreement Declarations Exclusions Solution: c 526. Deductibles are used for all of the following reasons EXCEPT

d. To eliminate small claims. d. To reduce premiums. d. To reduce loss control efforts. d. To reduce moral and morale hazard Solution: c 527. Which of the following statements is (are) true with respect to endorsements and riders? i. Endorsements and riders are used to amend provisions of insurance contracts. ii. If the endorsement or rider conflicts with terms in the underlying contract, the endorsement or rider takes precedence unless it conflicts with the law. d. I only d. Both I and II d. Neither I nor II d. II only Solution: b 528. Jitendra has an expected income stream over the next 15 years at Rs 25 lakh. If the inflation adjusted discount rate comes to 3% then what is the human life value when there is a lump sum requirement of another 3 lakh to clear off his outstanding debt. (Assume the income stream per annum will remain at the same level for the next 15 years) d) Rs 1304654 d) Rs 1604654 d) Rs 1904654 d) Rs 2204654 Solution: c 529. Which one of the following is not come under principal merit rating method: ) Schedule rating ) Experience rating ) Retrospective rating ) Judgement rating Solution d Rating could be 1. Individual or judgmental 2. Class or manual 3. Merit rating Schedule , Experience & Retrospective 530. As per the Income Tax Act, 1961 if Annual Premium paid is more than 20% of Sum Assured, then (d) Such Premium is fully eligible for deduction u/s 80 C upto maximum of Rs. 1 Lakhs and the Maturity Value, Surrender Value is fully exempt u/s 10(10D). (d) Such Premium is eligible for deduction u/s 80 C upto maximum of Rs. 1 Lakhs but only to extent of the first 20% of the Sum Assured but Maturity Value, Surrender Value is not exempt u/s 10(10D) with the exception of only Death Benefit. (d) Both a & b are false (d) Only b is False (d) None of the above Solution: b 531. M/s XYZ Ltd is entering the Capital Market to raise fresh funds(IPO) for its expansion, this is an example of ____________ from M/s XYZ Ltd point of view. d. Risk Retention d. Risk Reduction d. Risk Sharing d. Risk Transfer d. A combination of all above d. None of the above Solution: c 532. Which of the following is the third step in Risk Management Strategy e) Identification of the Risks e) Develop Alternative for handling risks e) Choose and implement an appropriate Strategy. e) Analyze and Evaluate risks ( Risk Measurements) e) None of the above Solution: b 533. As per Sec 10(10D) of the Income Tax Act, any sum received from Insurance Company is exempt fully for Income Tax. (e) True (e) False (e) True only when Annual Premium paid is less than or equal to 20% of Sum Assured (e) True for Death Benefit without any reference to Annual Premium condition (e) Both C & D are true Solution: e 534. Premium paid to Life Insurance for Basic Sum Assured (Death Benefit) is eligible for deduction under which section of the Income Tax Act, 1961. (e) Sec 80 C upto a Maximum of Rs. 1 Lakh (e) Sec 80 C without any Maximum limit

(e) Sec 80 D (e) None of the above Solution: a 535. Premium paid to Life Insurance for Value Additions such as Critical Illness Rider is eligible for deduction under which section of the Income Tax Act, 1961. (d) Sec 80 C upto a Maximum of Rs. 1 Lakh (d) Sec 80 C without any Maximum limit (d) Sec 80 D upto a Maximum of Rs.15000 (d) Sec 80 D without any Maximum limit (d) None of the above Solution: c 536. Mr. A aged 22 years has just started working in a BPO earning Rs.20,000 per month and has no dependants and is not financially responsible to his family. As a Financial Planner recommend the appropriate Insurance Plan. (e) Term Insurance (e) Term Insurance with Personal Accident and Disability Insurance (e) Only Personal Accident and Disability Insurance (e) No Insurance required, he should invest all his savings in Wealth Creation Solution: c 537. Mr. Bhavesh aged 32 years the sole breadwinner with dependant Spouse and one child named Asha aged 3 years. As a Financial Planner recommend the appropriate Insurance Plan. (Note Asha will complete her MBA by her 25 age) d) Term Insurance with Critical Illness Rider, Disability Rider & Waiver of Premium Rider for the next 22 years d) Term Insurance with Critical Illness Rider, Disability Rider with Waiver of Premium for the next 25 years d) ULIP d) Money Back Policy for 25 years d) Endowment Policy for 22 years d) No Insurance required, he should invest all his savings in Wealth Creation for Asha MBA Education d) None of the above Solution: a 538. Mr. Chitale, a practicing Chartered Accountant age 43 years provides Advisory Services relating to Mergers & Acquisitions, Corporate Debt Restructuring, Capital Financing. As a Financial Planner recommend the Priority Insurance Plan. (Note: Mr Chitale will retire at 60 age) g) Term Insurance with Critical Illness Rider, Disability Rider & Waiver of Premium Rider for the next 17 years g) ULIP g) Money Back Policy for 17 years g) Endowment Policy for 17 years g) Professional Indemnity Insurance g) None of the above Solution: e Risk and Insurance 82 539. As per the Insurance Act, 1938 an Insurance Company has the right to question the claim on the basis of concealment of facts, mis-statements and the like in the proposal form, such right is covered under which clause? f) Incontestability Clause u/s 45 covering only the first two years form the policy commencement date f) Incontestability Clause u/s 45 covering only the first three years form the policy commencement date f) Incontestability Clause u/s 45 covering only the first five years form the policy commencement date f) Refusal of Claim clause u/s 45 covering only the first two years form the policy commencement date f) None of the above Solution: a 540. Mr. Rajesh is worried about its child (Sunita aged 5 years) education expenses and he does not want to stop her child education due to lack of funds, As a Financial Planner recommend the Insurance Plan which will meets the need of security for childrens education e) Term assurance plan covering period till the child completes her Professional Education e) Children Deferred Assurance Plan e) Deferred annuity plan e) Health insurance e) ULIP e) None of the above Solution: b 541. Industrial Plant Insurance Policy has a Franchise Clause of Rs. 50,000, the claim filed is Rs.70,000. What would be the claim amount payable by the insurance company under the said policy? ) Rs. 50,000 ) Rs. 70,000 ) Rs. 20,000 ) Rs. 1,20,000 ) None of the above Solution: b 542. Boiler Plant Insurance Policy has a Franchise Clause of Rs.30,000, the claim filed is Rs. 29,000 What would be the claim amount payable by the insurance company under the said policy? e) Rs. 30,000

e) Rs. 29,000 e) Rs. 59,000 e) Rs. Nil e) None of the above Solution:d Franchise is a type of deductible where in claim above franchise limit is fully paid and below which nothing is paid. 543. In Mediclaim, what is the maximum amount of premium which is deductible under The Income Tax Act, 1961 e) Rs.10,000, only if paid by a cheque Sec 80 DD e) Rs.15,000, only if paid by cheque Sec 80 D e) Rs.15,000, only if paid by a cheque Sec 80 DD e) Rs.10,000, only if paid by a cheque Sec 80 D e) None of the above Solution: bFranchise is a type of deductible where in claim above franchise limit is fully paid and below which nothing is paid 544. The immediate Cash needs of an individual will come to Rs 30000 for the funeral (final cost) plus Rs 3 lakh as outstanding debt.. The Net Income needs at a present value are Rs 4 lakh. With an existing asset base of Rs 2 lakh, As a Financial Planner compute the insurance required under Needs Approach? f) Rs 330000 f) Rs 530000 f) Rs 730000 f) Rs 930000 f) None of the above Solution:b 4 + 3 + 0.30 2 = 5.30 lakhs 545. An Insurance planner and prospect agreed for an endowment policy for Rs.8 lakhs limited premium payment for 10 years with a term of 20 years. If revisionary bonus is taken as 7.5 % per annum and terminal bonus as Rs. 150 per Rs.1000 sum assured, what will be the Maturity value? e) Rs.15,20,000 e) Rs.21,20,000 e) Rs.13,20,000 e) Rs.20,00,000 e) None of the above Solution:b Revisionary bonus = 800000*7.5 / 100 = 60000 * 20 = 1200000 Terminal bonus = 150 * 800 = 120000 Maturity value = 8 + 12 + 1.2 =21.2 lakhs 546. The following are statements made concerning contracts of insurance. Identify the statement/s that is/are correct. (i) For life insurance contracts, misstatement of the insureds age constitutes a voidable misrepresentation. (ii) An innocent misrepresentation by an applicant for insurance constitutes fraud. e) only (i) is correct e) only (ii) is correct e) Both (i) & (ii) are correct e) Neither (i) or (ii) are correct Solution: d 547. The maxim buy term and invest the balance may not be a feasible proposition for many prospects for the following reasons EXCEPT for: d) it may be difficult to achieve sufficient diversification in the invested assets d) it may be difficult to achieve a suitable investment portfolio with the desired risk reward relationship d) it may be possible to consistently outperform the investment returns earned by an established life office d) that the insurer offers capital guarantees on cash values d) None of the above Solution: c 548. An individual may prefer investment-linked assurance to conventional assurance policies for the following reasons EXCEPT e) he has some direction over the investment of his premiums e) he is informed of the expenses charged for the services provided e) he is attracted by the guaranteed surrender values offered under these contracts e) he prefers the switching facilities available under such contracts e) None of the above Solution: c 549. Which of the following does not constitute a valid charge on the premiums paid for an investment-linked life assurance policy? e) bid-offer spread e) front-end charges e) recurrent fund related charges e) surrender penalty

e) None of the above Solution: d 550. Mr. Ankush is in very good health without any background of existing diseases, he needs a temporary assurance Cover, as a Financial Planner recommend an appropriate insurer? e) Insurer with a stringent underwriting policy e) Insurer with a lax underwriting policy e) Insurer with very few causes of death excluded e) Insurer with extensive causes of death excluded e) None of the above Solution: a 551. Identify the class of assurance for which an insurer, in a costing exercise, must provide for future anticipated improvements in mortality d) investment-linked endowment assurance d) conventional endowment assurance d) group life assurance d) life annuities d) none of the above Solution: d 552. Mr. Suresh has come with the following risk matrix, which he is exposed to, As a Financial Planner advise him an appropriate Risk Management Strategy. Sr.No. Frequency of Event Severity of Financial Loss 1. Low High 2. High High 3. Low Low 4. High Low e) Risk Transfer, Risk Avoidance, Risk Retention & Risk Reduction e) Risk Avoidance, Risk Control, Risk Retention & Risk Reduction e) Risk Transfer, Risk Retention, Risk Avoidance & Risk Reduction e) Risk Transfer, Risk Reduction, Risk Avoidance & Risk Retention e) None of the above Solution: a 553. Professional Indemnity Insurance policy protects a financial planner who has been negligent in giving investment advice: e) Only if the financial planner is liable under statute e) Only if a contract exists between the financial planner and the investor e) Only if the investor relies on the advice e) Only if the financial planner has not included a disclaimer clause in the contract with the investor e) Only if the financial planner is a Qualified Certified Financial Planner e) None of the above Solution: c 554. 'Consideration' under the law is a written promise to ________ (i) Do certain things (ii) Abstain from doing certain things (iii) Forbear some acts (iv) Accept an offer f) (i), (ii), and (iii) only f) (i), (ii), and (iv) only f) (i), (iii), and (iv only f) (ii), (iii), and (iv) only f) None of the above Solution: a 555. Mr. Frank has an Indemnity Policy in respect of his owned house. The house was valued when he first took out the policy three years ago at Rs.80,000 and insured for that amount. The policy has been renewed each year since, without any alteration or modifications. In the current year, the said house was completely destroyed by fire and the cost of rebuilding it was Rs.110,000 (Market Value) plus he paid Rs. 35,000 to his neighbour whose house was accidentally burnt to some extent due to the fire. As a Financial Planner compute his claim amount which will be paid by the Insurance Company. e) Rs.58,182 e) Rs.80,000 e) Rs. 93,182 e) Rs.1,45,000 e) None of the above Solution: a 80000 * 80000/ 110000 = 58182. 556. Mrs. Parab explains that she needs a life insurance policy that will cover her family against financial risk over the next five years, while she still has dependent children and a large home mortgage. It is unlikely her income will increase over the covered tenure. As a Financial Planner advise her about her Insurance Policy. e) Children Benefit Policy for 5 years e) Term insurance with a level premium for 5 years

e) Single premium Money Back Policy maturing after 5 years e) Mortgage Redemption Insurance for 5 years e) Whole of life assurance to be made paid up after five years e) ULIP to be redeemed after 5 years e) None of the above Solution: b 557. Mr. Sanjay aged 59, presently in his retirement phase, has maintained a high level of life insurance, the reason being? (i) Pay estate duty (ii) Provide succession or dependent income (iii) Repay outstanding debt (iv) Build cash value g) Only (i) and (ii) g) Only (iii) g) Only (i), (ii) and (iii) g) All of the above g) None of the above Solution: b 558. Terminal Bonus is to be paid ________________ e) Compulsory by the Insurance Company as per the Insurance Act, 1938 e) Is paid as a % of sum assured every year e) The Insurance Company usually pays it voluntary for being disciplined in regular premium payment e) None of the above Solution: c 559. Shelar & Suvarna are married couple who are 44 & 40 years respectively. They want a regular series of payments in their Sunset Years covering both of their lives. Shelar wish is that Suvarna should get the regular payment even if he becomes a departed soul , the same is the wish of Suvarna, that Shelar should get the regular payment even if she becomes a departed soul. As a Financial Planner address their concerns by giving them an appropriate advise, i.e. the couple should select? e) Life time Annuity e) Immediate Annuity e) Deferred Annuity e) Annuity Certain e) Annuity Certain with Life Time Annuity e) Joint-and-Survivor Annuity e) None of the above Solution: f 560. Rupali purchased a Health Insurance. The policy has a calendar-year deductible of Rs.500 and 80:20 coinsurance. Rupali was hospitalized with a covered illness on January 23rd. This hospitalization was her first claim under the said policy for the calendar year. Her covered medical expenses were Rs.20,500. How much of this amount will the insurer pay, and how much will Rupali be required to pay to the Hospital? g) The insurer will pay Rs.16,500 and Rupali will pay Rs.3,500 g) The insurer will pay Rs.20,500 and Rupali will pay Rs.Nil g) The insurer will pay Rs.15,500 and Rupali will pay Rs.4,500 g) The insurer will pay Rs.20,000 and Rupali will pay Rs.500 Solution: d 561. Mr. Praful aged 20 , wants a regular series of payments of Rs. 1,200 per annum in accrued form with an escalation of 2% per annum from his age 55 to next 20 years without any interruption. If the annual rate of earnings is taken as 8% per annum compounded annually throughout the working life & retirement life and an Inflation Rate of 5% per annum during his golden years. As a Financial Planner find what contribution he is required to make right now in accrued form if he wishes to make a Single payment (Ignore impact of Taxes) d) Rs. 1459.25 d) Rs. 1458.25 d) Rs. 1460.25 d) Rs.1898.75 d) None of the above Solution: a Inflation adjusted return = 2.86 %.But the cost of living increasing by 2% and hence effective increase would be 0.86 %. Find pv given pmt = 1200 ; rate = 0.86% nper = 20 years. Pv = 21963. Now find pv given nper=35 rate = 8 % fv = 21963 pv = 1485 562. Mr. Ramesh aged 20 , wants a regular series of payments of Rs. 1,200 per annum in accrued form from his age 55 without any interruption where he want its certain for the first 10 years and thereafter if he survives he wants the same till he becomes a departed soul. To achieve the aforesaid retirement goal he is ready to invest in a Single payment a maximum of Rs.2500 now in accrued form, Mr. Ramesh left for heavenly abode on his 75th birthday. If the annual rate of earnings is taken as 6% per annum compounded annually throughout the working life & sunset years. As a Financial Planner find out the IRR(Ignore impact of Taxes) e) 4.22 %

e) 2.22 % e) 3.22 % e) 2.82 % e) None of the above Solution:b find fv given rate = 6% ; nper = 35 ; pv = 2500 ; fv= 19215 Now find rate given pv=19215; nper = 20; pmt = 1200 IRR=2.22 % 563. Mr. Shreekant aged 30 years took a Pure Risk Cover (Term Insurance) bearing a sum assured of Rs.10 Lakhs for a term of 25 years costing an annual premium of Rs. 3,500 per annum. After paying the said premium for a period of 20 years well on time, he wants to surrender the policy. (Note:- Its the Insurance Company official standard to pay Surrender Value @ 75% of Accumulated Premium if the policy has been serviced for 19 years and more) As a Financial Planner find out the Surrender Value of the said policy if any? e) Rs. Nil e) Rs. 52500 e) Rs. 49,875 e) Rs. 70,000 e) None of the above Solution: a 564. Co-Insurance means _________ (i) Loss is shared in proportion of Insurance (ii) The goal is to provide equity in premium payment (iii) The Insured Co. will again insure it with some other Insurer. e) Both (i) & (ii) e) Only i e) Only ii e) All of the above e) None of the above Solution: a 565. Maggi & Savio are co-applicants of a mortgaged house,They are on the verge of a divorce, however they are spectical about the action that may be initiated by the Housing Finance Company (HFC) once they are divorced. As a Financial Planner address their concerns by giving them an appropriate advise about the action of the Home Loan Lender. e) HFC will not interfere as long as the EMIs are being paid on time e) HFC will repossess the house after divorce e) HFC will insist on the house being transferred to only one of them e) HFC will appoint a Counselor for reconciliation between the couple. e) HFC will increase the interest rate in order to compensate for the increased risk e) HFC will apply the Forfeiture Clause immediately on divorce e) None of the above Solution: a 566. Edward a Roman Catholic got married on 25th December 2005 in the Marriage Court under Special Marriages Act. On 28th December 2005 does his wife Anita a Hindu have an insurable interest in the life of Edward ? (Note :- Anita after Marriage wont get converted to Christianity) g) No, Insurable Interest is acquired only after a year of marriage g) Yes, Anita has an automatic insurable interest in the life of Edward soon on marriage and the likewise for Edward g) No, only Edward has an insurable interest on the life of the Anita and not the other way round. g) No, Insurable Interest is only among the same caste g) None of the above Solution: a 567. The proceeds of Rs.150,000 from a life insurance policy is to be disbursed under a settlement option using an annuity accrued with term certain of 15 years. Assuming that a 6% per annum interest compounded annually is appropriate, As a Financial Planner find out the level annual payments arising under this arrangement e) Rs.13,491 e) Rs.12,972 e) Rs.15,444 e) Rs.14,570 e) None of the above Solution: c find pmt given pv = 150000; rate 6 % ; nper = 15 568. The proceeds of Rs.250,000 from a life insurance policy is to be disbursed under a settlement option using an annuity accrued with term certain of 15 years. Assuming that a 8% per annum interest compounded monthly is appropriate, As a Financial Planner find out the level annual payments arising under this arrangement e) Rs.2373

e) Rs.2389 e) Rs.27043 e) Rs.29207 e) None of the above Solution: b find pmt given pv = 250000 rate = 8 % /12 and nper 15 * 12 569. Rupesh aged 33 and Sunita aged 29 (Married Couple) approach you a Financial Planner with the following data:Funds for Investment are Limited & Needs are in ample as follows:5) To start an investment plan for funding the Education of their child Akshay age 3 5) To set up a Testamentary Trust for their child; 5) To set up a contingency fund amounting to 3 months of living expenses 5) To start saving for retirement; 5) To purchase life and health insurance policies. As a Financial Planner address their concerns by giving them an appropriate advice by sequencing there needs in the order of priority. e) 3,5,1,4,2 e) 5,1,2,4,3 e) 5,3,1,4,2 e) 3,5,2,1,4 e) None of the above Solution: a 570. Mr. Andrew recently purchased a 2 BHK Flat worth Rs. 30 Lakhs and wants to Insure it, As a Financial you advise him yes, the flat can be insured by giving the following reason __ e) Insurable interest has been created by Common Law e) Insurable interest has been created by Contract e) Insurable interest has been created by Statute e) Insurable interest has been created by Ownership e) None of the Above e) All of the Above Solution: d 571. In the event of a claim, an Insurance contract governed by Indemnification clause pays to the insured a claim amount equal to ________ f) the sum insured f) the extent of loss admitted subject to a maximum equal to the sum insured f) the extent of loss admitted without any limits f) a previously agreed amount f) Negotiable amount between the Insured & Insurer f) None of the above Solution: b 572. Mr. Sunil has an Endowment Life Insurance Policy where he has duly paid his premiums for the past 15 years out of a term of 20 years, he is in urgent needs of funds only for a short period in order to marry off his son Paras, As a Financial Planner advise him the maximum loan he can avail on the said policy ? f) 85% of Surrender Value f) 90% of Paid up Value f) 90% of Surrender Value f) 90% of all premiums paid f) 75% of all premiums paid f) None of the above Solution: c 573. The characteristics of Insurable Risks are :6. Large Number of Homogenous Units 6. Definite and Measurable 6. Fortuitous or accidental 6. Not Catastrophic d) 1 & 2 d) 2, 3 & 4 d) 1,2 & 3 d) 1,2,3 & 4 d) None of the above Solution: d 574. Mr. Praful has given his data as follows, Current Age 35 years, Will retire at 55 Retirement Fund :- Required 2/3 of his last drawn Salary per annum in due form Life expectancy 20 years on retirement His current annual salary is Rs. 60,000 and this escalates by 6% per annum throughout his working career, His Employer gives the annual escalations on his birthdays.

Assume the rate of earning of 8% per annum compounded annually throughout the overall period, As a Financial Planner answer Question No. 61, 62 & 63 575. Calculate his last drawn annual salary on the eve of retirement e) Rs.128,400 e) Rs.181,536 e) Rs.192,428 e) Rs.132,000 e) None of the above Solution: b find fv given pv = 60000 rate = 6% nper 19 fv= 181536 576. Calculate the present value of the retirement income as on the Current age of 35 yrs e) Rs.2,25,800 e) Rs.2,54,933 e) Rs.2,75,328 e) Rs.3,05,780 e) None of the above Solution: c 181536 * 2/ 3 = 121024 is pmt rate 8% nper 20 pv= 1283290 Again find pv given fv= 1283290 rate 8% nper 20 pv= 275328 577. Calculate the level annual amount that he must deposit during his working life, assuming that the first is due now, to enjoy the stated retirement income during his sunset years. e) Rs.28,042 e) Rs.25,965 e) Rs.24,042 e) Rs.28,890 e) None of the above Solution: b find pmt given fv= 1283290 rate = 8% nper = 20 and type 1 578. Participating ordinary life policies with a sum assured of Rs. 10,000 are issued by two Life Insurance Companies, Lic and I-Pru, to Suresh aged 35 with the following cost data: Data LIC (Rs.) I-Pru (Rs.) Annual premiums 230 290 Accumulated value of Total bonuses for 20 years at 6% pa 1613 1700 Cash value at end of 20 years 3620 4000 Based on the above information and assuming a 6% per annum interest rate, the annual surrender cost index for each Rs.1,000 sum assured at the end of 20 years ____ e) For Lic is greater than I-Pru e) For Lic is equal to I-Pru e) For I-Pru is greater than Lic e) Is not determinable, since data is insufficient e) None of the above Solution: c 579. Ms. Kavita has an Endowment Life Insurance Policy where she has duly paid the premiums, she is in urgent needs of funds which she doesnt want to repay back in order to marry off her only daughter Sanjivani, As a Financial Planner advise her the maximum amount she can avail on the said policy on its due surrender? Date of commencement 5th October, 1990 Date of last premium 5th April, 2000 Date of birth 1st May, 1965 Mode of premium payment Half Yearly Premium Rs. 20 per annum for Rs. 1000 sum assured Plan Endowment with profit Term 25 years Sum assured Rs.1, 20,000 Bonus accrued from 31st March 1991 to 31st March 2000 is Rs.70 per Rs. 1,000 sum assured Accidental benefit Rs.2 per annum for Rs. 1000 sum assured S.V. factor 75% of Accumulated Value e) 24000 e) 81,810 e) 80,810 e) 80,100 e) None of the above Solution: e paid up value = 120000*21/50 = 50400 bonus 70*120*10 = 84000 Accumulated value = 134400 * 75/100 = 100800 is SV. 580. Lic Premium Table depicts the following data for a certain policy in which your client Mr. Sanjay is interested Tabular premium:

Rs.33.10 per Rs. 1000 sum assured. Rs.2 less for yearly mode. Re.1 less for half yearly mode. Rs.3 less for sum assured of Rs.1, 00,000 and above. Double accident is allowed up to a maximum of Rs.10 lakhs sum assured on payment of Re.1 per 1000 sum assured. As a Financial Planner calculate the yearly premium for Rs.15 lakhs sum assured with occupation extra of Rs.4 per thousand sums assured e) Rs. 49,150 e) Rs. 48,150 e) Rs. 48,650 e) Rs. 52,350 e) None of the above Solution: b 33.10 2 3 + 4 = 32.1 * 1500 = 48150 581. Mr. Jagadish has given his personal details as follows:Current Age 30 yrs , Plans to Retire at age 65. Job Profile :- Senior Manager in Telco Ltd with Annual Salary of Rs.10,00,000 Annual Cash Outflow Professional tax of Rs.5,000 Income tax Rs. 1,95,000 Reasonable self-maintenance expenditure Rs.1,00,000 p.a. Life insurance premium for self Insurance Rs. 20,000 having sum assured Rs. 20 Lakhs Life insurance premium for Sulekha (wife) Rs. 13000 having sum assured Rs. 5 Lakhs Life insurance premium for Aditya (son) Rs. 7000 having sum assured Rs. 2 Lakhs Assume:- Rate of interest for capitalization of future income is at 10%. As a Financial Planner recommend the Insurance Cover using HLV Method e) Rs.46 lakhs e) Rs.68 lakhs e) Rs.48 lakhs e) Rs.66 lakhs e) None of the above Solution: b10.00 3.20 [ 2+ 1+ 0.2] = 6.80 lakhs find pv given pmt = 6.8 lakhs; rate = 10 % ; nper=35 pv= 72 lakhs 20 lakhs [present cover]= 52 lakhs 582. Mr. Shelar invests Rs.5000 in a Bank Deposit today @ 8% p.a compounded monthly. He hopes that this nvestment will enable him to fund his college education (estimated to cost Rs.9000) which commences after 4years. As a Financial Planner compute the end value of this investment after four years? (Ignore the impact of Inflation & the rise in Education cost) e) Rs. 6802 e) Rs. 6870 e) Rs. 6878 e) Rs. 6925 e) None of the above Solution: c find fv given pv = 5000; rate 8% /12 and nper 48 583. Kavita invests Rs.5000 per year at the beginning of each year for 5 years @ 5% p.a. in a bank deposit. She then withdraws the accumulated sum over a period of 3 equal annual installments at the end of the said each period. As a Financial Planner compute the value of the deposit at the end of 5 years and the quantum of withdrawal each year thereafter? e) Rs.27628, Rs. 10145 e) Rs. 29010, Rs. 10145 e) Rs. 29568, Rs. 11054 e) Rs. 29010, Rs. 10653 e) None of the above Solution: d Find fv given rate= 5 %; nper = 5 pmt=5000 type =1 fv = 29010 Now find pmt given pv= 29010 ; rate = 5 % ; nper = 3 ; Pmt = 10653 584. Mr. and Mrs. Noronha, aged 40 and 36 years, both have a life expectancy of another 40 years. Their Data Sheet depicts the following information: Mr. Noronha is the sole breadwinner of the family and the family has no children Current investments has a Market Value of Rs.20,00,000 Annul Expenses Rs.4 Lakhs (including Rs. 1 Lakh of Mr. Noronha personal Exps)

Mr. Noronha income post tax Rs. 3.5 lakhs. Final costs Rs. 1 lakhs (Funeral Exps of Mr. Noronha). Post tax, post inflation rate/discount factor is 3 % per annum As a Financial Planner find out the Insurance Requirement of Mr. Noronha as per Needs Approach & Income Replacement Approach (Ignore Impact of Inflation) e) 69.34 lakhs, 83.33 lakhs e) 49.34 lakhs, 68 lakhs e) 50.34 lakhs, 83.33 lakhs e) 70.34 lakhs, 83.33 lakhs. e) None of the above Solution:c Need based method for the life cover : 52.42 lakhs pmt = 3 lakhs ; nper = 40 ; rate = 3 % pv= 71.42 lakhs Add final cost [1] and deduct investments[20] =52.42 lakhs Income replacement = 2,50,000 * 100 / 3 = 83.33 lakhs 585. Mr. Rohan has approached you for his Life Insurance Needs, As a Financial Planner which of the following data will be required by you to calculate his Life Insurance Needs as per Human Life Value Method 6) Self Life Insurance Premium with its Sum Assured 6) Self Profession Tax & Self Income Tax paid 6) Income earned by his wife Renuka as well as her Life Insurance Sum Assured and its Premium 6) Income earned by his Child Ajay age 12 6) Income received by Rohan from HUF since being a Co-member of such HUF 6) Average Annual Earnings during his entire working career 6) Appropriate Capitalization rate 6) Present Age, Years of Working Life planned ) Only 1, 2, 6, 7 & 8 ) Only 1,2,3,5,6,7 & 8 ) All except 3 & 4 ) All of the above Solution: a 586. Raman needs a life insurance policy for Rs 5 lakh that will require a premium commitment of Rs 23000 each year for the next 15 years. With an earning potential of 5% the adjusted interest earning potential comes to a) Rs. 396406 b) Rs. 496306 c) Rs. 596406 d) Rs. 696406 Solution: b find fv given nper = 15 ; rate = 5 % ; pmt = 23000 As one pays premium as advance this is type =1 Fv = 521122 587. A person requires Rs 50000 per annum for the 15 years after retirement, which is still 10 years away. With a discount rate of 5% the need for the insurance based on this parameter is a) Rs. 245675 b) Rs. 318610 c) Rs. 456376 d) Rs. 567645 Solution: b find pv given pmt=50000; rate = 5% ; nper = 15 Now fv = 544932 ; rate = 5 % nper = 10 pv= 334541.

Vous aimerez peut-être aussi